Как найти силу трения по горе

Условие задачи:

С горы высотой 2 м и основанием 5 м съезжают санки, которые затем останавливаются, пройдя по горизонтали путь 35 м от основания горы. Найти коэффициент трения.

Задача №2.3.16 из «Сборника задач для подготовки к вступительным экзаменам по физике УГНТУ»

Дано:

(H=2) м, (L=5) м, (S=35) м, (mu-?)

Решение задачи:

Схема к решению задачиРешать эту задачу будем, используя закон сохранения энергии. Согласно этому закону, работа силы трения равна изменению полной механической энергии.

[A = Delta E;;;;(1)]

Когда санки в конце концов остановятся, то их полная механическая энергии станет равной нулю, хотя в начале они имели потенциальную энергию (mgH). Изменение энергии (Delta E) равно:

[Delta E = 0 – mgH]

Так как сила трения принимает разные значения при движении саней по горе и горизонтальной поверхности, то работу будем находить как сумму работ:

[A = {A_1} + {A_2}]

Таким образом равенство (1) примет такой вид:

[{A_1} + {A_2} =  – mgH;;;;(2)]

Рассмотрим участок движения по горе. Из первого закона Ньютона в проекции на ось (y) найдем силу реакции опоры:

[{N_1} = mg cdot cos alpha ]

Тогда сила трения скольжения на этому участке равна:

[{F_{тр1}} = mu N_1]

[{F_{тр1}} = mu mg cdot cos alpha ]

Работа силы трения отрицательна, так как сила направлена против перемещения, и равна:

[{A_1} =  – {F_{тр1}} cdot l]

[{A_1} =  – mu mg cdot l cdot cos alpha ]

Интересно, но из прямоугольного треугольника следует, что:

[L = l cdot cos alpha ]

Тогда:

[{A_1} =  – mu mgL;;;;(3)]

Аналогично определим работу силы трения на участке движения по горизонтальной поверхности, объяснения опустим.

[{N_2} = mg]

[{F_{тр2}} = mu mg]

[{A_2} =  – {F_{тр2}} cdot S]

[{A_2} =  – mu mg cdot S;;;;(4)]

Подставим (3) и (4) в (2), тогда:

[ – mu mg cdot L – mu mg cdot S =  – mgH]

Сократим обе части равенства на (mg) и домножим на (-1), далее уже выразим искомый коэффициент (mu).

[mu  = frac{H}{{L + S}}]

Вычислим ответ:

[mu  = frac{2}{{5 + 35}} = 0,05]

Ответ: 0,05.

Если Вы не поняли решение и у Вас есть какой-то вопрос или Вы нашли ошибку, то смело оставляйте ниже комментарий.

Смотрите также задачи:

2.3.15 С каким ускорением движутся грузы m1=0,5 кг и m2=0,6 кг, если высота наклонной
2.3.17 Чему должен быть равен минимальный коэффициент трения между шинами
2.3.18 Два бруска одинаковой массы 0,2 кг поставили на наклонную плоскость с углом

Благодаря этой силе автомобили тормозят на светофоре, катер останавливается в воде, колесо буксует в яме. Как вы уже поняли, в этой статье мы будем разбираться, как решать задачи на силу трения.

Сила трения имеет электромагнитную природу. Это значит, что эта сила проявляется в результате взаимодействия частиц, из которых состоит вещество.

Хотите больше полезной и интересной информации по разным темам? Подписывайтесь на наш телеграм-канал.

Что нужно знать о силе трения, чтобы решать задачи

Трение – один из видов взаимодействия тел, которое возникает при их соприкосновении.

Сила трения всегда направлена в сторону, противоположную движению и по касательной к соприкасающимся поверхностям. Между твердыми телами возникает сухое трение, а при движении тел в жидкостях или газах говорят о вязком трении.

Что нужно знать о силе трения, чтобы решать задачи
Природу этой силы мы уже установили. Помимо этого нужно знать, что бывают разные виды сил трения:

  • трение покоя; 
  • трение скольжения;
  • трение качения (при перекатывании тел друг по другу);
  • сопротивление среды (для движения в жидкости).

Вот пример на виды силы трения: брусок лежит на столе, и никто его на трогает. В этом случае действуют только сила тяжести и сила нормальной реакции опоры. Если мы начнем толкать брусок, но так сильно, чтобы его сдвинуть, на него будет действовать сила трения покоя, по третьему закону Ньютона равная внешней силе, приложенной к бруску. Сила трения покоя имеет предельное значение. Если внешняя сила будет больше этого значения, брусок начнет скользить по столу. В этом случае говорят о силе трения-скольжения. А вот и простейшая формула для силы трения:

Что нужно знать о силе трения, чтобы решать задачи

«Мю» — коэффициент трения скольжения. Это безразмерная величина, которая зависит от материалов взаимодействующих тел и от качества их поверхностей. Величина коэффициента трения не превышает единицы.

При решении простых физических задач силу трения скольжения часто принимают равной максимальной силе трения покоя.

Не забывайте также про нашу памятку и подборку полезных формул.

Вопросы по теме «Сила трения»

Вопрос 1. От чего зависит сила трения?

Ответ. Взглянем на формулу выше, и ответ придет сам. Сила трения зависит от свойств соприкасающихся тел, силы нормальной реакции опоры, скорости относительного движения тел.

Вопрос 2. Зависит ли сила трения от площади соприкасающихся поверхностей?

Ответ. Нет, площадь не влияет на силу трения.

Вопрос 3. Какими способами можно уменьшить или увеличить силу трения?

Ответ. Можно уменьшить коэффициент трения, сделав сухое трения вязким. Для увеличения силы трения необходимо увеличить давление на них.

Вопрос 4. Тело покоится на плоскости. Действует ли на него сила трения?

Ответ. Если на тело не действуют внешние силы, то сила трения покоя, по третьему закону Ньютона, равна нулю.

Вопрос 5. Какая из этих сил самая большая по модулю: сила трения покоя, сила трения качения или сила трения скольжения?

Ответ. Сила трения скольжения имеет самое большое значение.

Вопрос 6. Какие есть примеры полезного действия силы трения?

Ответ. Среди полезного использования силы трения можно выделить работу тормозов транспортных средств, добычу огня первобытными людьми.

Задачи на силу трения с решениями

Кстати! Для наших читателей действует скидка 10% на любой вид работы.

Задача №1. Нахождение силы трения

Условие

Брусок массой 5 килограмм скользит по горизонтальной поверхности. Сила трения скольжения равна 20 Н. Найдите силу трения, если масса бруска уменьшится в два раза, а коэффициент трения останется неизменным. 

Решение

Применим формулы:

Задача №1. Нахождение силы трения
Ответ: 10 Н.

Задача №2. Нахождение коэффициента трения

Условие

Тело скользит по горизонтальной плоскости. Найти коэффициент трения, если сила трения равна 5 Н, а сила давления тела на плоскость – 20 Н.

Решение

Сила давления тела на плоскость равна силе нормальной реакции опоры. 

Задача №2. Нахождение коэффициента трения

Ответ: 0,25

Задача №3. Нахождение силы трения и коэффициента трения

Условие

Лыжник массой 60 кг, имеющий в конце спуска скорость 10 м/с, останавливается через 40 с после окончания спуска. Определите силу трения и коэффициент трения.

Решение

Сначала найдем ускорение, с которым движется лыжник. Затем по второму закону Ньютона найдем силу, которая действует на него:

Задача №3. Нахождение силы трения и коэффициента трения

Ответ: 15 Н; 0,025.

Задача №4. Нахождение силы трения

Условие

Брусок массой 20 кг равномерно перемещается по горизонтальной поверхности под действием постоянной силы, направленной под углом 30° к поверхности и равной 75 Н. Каков коэффициент трения между бруском и плоскостью?

Решение

Сначала воспользуемся вторым законом Ньютона, учитывая, что ускорение равно нулю. Затем найдем проекции силы на вертикальную и горизонтальную оси:

Задача №4. Нахождение силы трения

Ответ: 0,4

Задача №5. Нахождение силы трения покоя

Условие

Ящик массой 10 кг стоит на горизонтальном полу. Коэффициент трения между полом и ящиком равен 0,25. К ящику в горизонтальном направлении прикладывают силу 16 Н. Сдвинется ли он с места. Какова сила трения между ящиком и полом?

Решение

Вычислим максимальную силу трения покоя:

Задача №5. Нахождение силы трения покоя

Так как приложенная сила по условию меньше, чем максимальная сила трения покоя, ящик останется стоять на месте. Сила трения между полом и ящиком, по третьему закону Ньютона, равна приложенной силе.

Ответ: 16 Н.

Нужна помощь в решении задач или других заданий? Обращайтесь за ней в профессиональный студенческий сервис.

  1. Сила трения в повседневной жизни
  2. Виды трения: примеры
  3. Законы трения
  4. Сила трения: формула/a>
  5. Как найти силу опоры?/a>

Некоторые физические явления могут казаться слишком сложными для понимания, особенно если изучать их только с научной точки зрения. К списку терминов, часто вызывающих много вопросов, относят в частности силу трения. Специально для вас подготовили материал, в котором попытались простыми словами объяснить это явление. Итак, разберем от А до Я, что такое трение, и как мы можем сталкиваться с ним в повседневной жизни.

Физика считается одним из самых сложных предметов школьной программы. Одно дело изучить все формулы наизусть, однако пониматься в них – задача не из простых. Неудивительно, что большинство школьников и студентов высших учебных заведений нуждаются в дополнительных занятиях, чтобы усвоить учебный материал.

Если вы хотите улучшить свои знания по физике, рекомендуем не медлить и просто обратиться за помощью к репетитору. Для этого воспользуйтесь сайтом БУКИ – супер удобной платформой для поиска преподавателей по любым предметам.

Сила трения в повседневной жизни

Еще на уроке физики в 7 классе мы изучили, что трение – это сила между двумя поверхностями, которые скользят или пытаются скользить друг на друге. Звучит немного сложновато, поэтому лучше объяснить на примере.

Что такое трение? В природе это физическое явление отвечает за множество процессов, которые мы наблюдаем каждый день. Мы ходим по земле, белки прыгают на ветках деревьев, вода точит камень, снежная лавина спускается с горы, машина едет по дороге – все это и есть сила трения.

Как действует трение?

Трение всегда действует в направлении, противоположном тому, в котором пытается двигаться объект. То есть трение всегда замедляет передвижение движущегося объекта. Величина трения зависит от материалов, из которых сделаны две поверхности. Чем шершавее поверхность, тем больше будет сила трения.

Хотя две контактирующие поверхности могут казаться очень гладкими, в микроскопическом масштабе они имеют много неровностей и впадин, которые приводят к трению. На практике невозможно создать предмет, имеющий совершенно гладкую поверхность.

Согласно Закону о сохранении энергии ни одна энергия в системе никогда не уничтожается. В большинстве случаев трение производит тепловую энергию, рассеиваемую через среду и сами предметы. Проверить утверждение, что трение производит тепло, достаточно просто. Попытайтесь быстро потереть руки. Вы почти сразу почувствуете, что они станут теплее ваших движений.

Трение – чрезвычайно полезная сила. Оно предотвращает скольжение нашей обуви по поверхности земли во время ходьбы, а также предотвращает скольжение автомобильных шин по дороге. Иногда мы хотим уменьшить силу трения. Например, мы используем масло, чтобы уменьшить трение между движущимися частями двигателя автомобиля, которые со временем могут выходить из строя из-за постоянного взаимодействия. Масло разделяет поверхность и может протекать между ними. Уменьшение трения означает, что движущиеся части автомобиля меньше изнашиваются и выделяется меньше тепла.

Читайте также: Как быстро выучить таблицу Менделеева?

Виды трения: примеры

Выделяют 3 вида трения: сила трения скольжения, сила трения покоя и сила трения качения. Расскажем о каждом из видов в упрощенном варианте, а также рассмотрим несколько простых примеров. 

Сила трения скольжения

Сила трения покоя

Сила трения качения

Это сила, возникающая при скольжении одного тела по поверхности другого.

Возникающая сила, когда пытаются сдвинуть одно из тел, препятствующее движению другого тела

Это сила, которая возникает, когда круглое тело катится по поверхности другого тела.

Пример:

Катание на санках, которое обожают все детишки. Производители санок создают их таким образом, чтобы поверхность ножек имела практически идеальную гладкость и они быстро спускались со снежных горок.

Пример:

Когда вам нужно передвинуть с одного места на другое тяжелый шкаф, диван или просто стул. Именно за такие процессы отвечает сила трения покоя. Сложность выполнения этих манипуляций состоит в том, что объект, который вы хотите сместить, находится в состоянии покоя, а вы пытаетесь сдвинуть его с помощью внешней силы.

Пример:

Движение автомобиля по дороге. Благодаря колесам машина может двигаться по дороге с бешеной скоростью и малыми энергетическими затратами.

Законы трения

С понятием трения классически отождествляют закон Амонтова-Кулона. Хотя впервые о силе трения рассказывал еще Леонардо да Винчи. В 1519 году он сформулировал утверждение, согласно которому трение – это процесс, возникающий при контакте одного тела с поверхностью другого тела. При этом сила трения должна быть пропорциональна нагрузке, направленной против направления движения.

Уже в 1699 году Амонтов открыл модель Леонардо да Винчи, а Кулон окончательно обосновал ее в своих работах. Для определения силы трения ученые ввели понятие коэффициента трения – физической константы, благодаря которой можно определить силу трения для любых контактирующих между собой материалов в условиях трения.

Читайте также: Что такое фотосинтез?

Сила трения: формула

Формула силы трения обычно выглядит так:

Fтр = k*N

що таке сила тертя

Где:

  • Fтр – это обозначение силы трения в физике. 
  • k – коэффициент трения. 
  • N – это сила сопротивления.

Сила трения измеряется в Ньютонах (Н).

Как найти коэффициент трения?

Как мы говорили ранее, коэффициент трения – это постоянная величина. В некоторых формулах его могут обозначать не буквой k, а символом µ. Определять коэффициент по отдельности не нужно, стоит лишь узнать значение в специальной таблице, где каждый материал имеет соответствующий показатель.

що таке сила тертя

Как найти силу опоры?

В отличие от коэффициента трения силу опоры нужно определять самостоятельно. Она имеет простую формулу:

N = m * g

  • N – это обозначение силы опоры в физике.
  • m – масса тела, действующего на поверхность другого тела.
  • g – ускорение свободного падения. Величина g постоянна. Она равна 9,8 м/с2.

Формула силы тяги

Для расчета силы трения качения используют несколько иную формулу – силу тяги. Сила тяги считается главной движущей силой любого транспортного средства. Она появляется в районе контакта авто шины и дорожного покрытия.

Сила тяги – это сила, прикладывающаяся на неподвижную поверхность для поддержания тела в постоянном движении. Силу тяги обычно находят с помощью второго закона Ньютона, в котором говорится что: сумма сил, влияющих на движущееся тело, равна массе, умноженной на ускорение.

Чаще силу тяги находят с помощью общей формулы:

Ft – Fc = m * a

  • Fт – сила тяги.
  • Fс – сила сопротивления.
  • m – масса тела.
  • a – ускорение. 

Ошибочно утверждать, что физика – это простая наука, которая по силам каждому. Во многих случаях, чтобы действительно понять сложную тему по этому предмету, недостаточно просто послушать урок или лекцию. Следует уделять много времени теории, а также на практике проверять каждую формулу, закон или аксиому.

Индивидуальные занятия – один из лучших способов изучить непонятный предмет, в частности, физику. Если вам нужен репетитор, вы можете найти его на нашей платформе БУКИ. Мы собрали более 90 тысяч учителей в одном месте, чтобы вы могли выбрать специалиста в соответствии с вашими потребностями и пожеланиями.

Читайте также: Тренируем память: советы для взрослых и детей

Содержание:

Сила трения и коэффициент трения скольжения:

Наблюдение: автомобиль после выключения двигателя через определённое время останавливается. Шайба, движущаяся по льду, также со временем остановится. Останавливается велосипед, если прекратить крутить педали.

Что же является причиной уменьшения скорости движения тел ?

Из ранее изученного вы знаете, что причиной изменения скорости движения тел есть действие одного тела на другое. Значит, в рассматриваемых случаях на каждое движущееся тело действовала сила. Тела остановились, поскольку на них в направлении, противоположном их движению, действовала сила, называемая силой трения Сила трения в физике - формулы и определения с примерами

Сила трения возникает между взаимодействующими твёрдыми телами в местах их соприкосновения и препятствует их относительному перемещению.

Одной из причин возникновения силы трения является шероховатость соприкасающихся поверхностей тел. Даже гладкие на вид поверхности тел имеют неровности, бугорки и царапины. На рисунке 81 эти неровности изображены в увеличенном виде.

Сила трения в физике - формулы и определения с примерами

Когда одно тело скользит по поверхности другого, эти неровности зацепляются одна за другую, что создает силу, затрудняющую движение. Вторая причина трения — взаимное притяжение молекул соприкасающихся поверхностей тел. Если поверхности тел очень хорошо отполированы, то их молекулы оказываются так близко друг от друга, что начинает заметно проявляться притяжение между ними. Различают несколько видов трения в зависимости от того, как взаимодействуют трущиеся тела: трение покоя, трение скольжения, трение качения.

Опыт 1. Положим брусок на наклонную доску. Брусок находится в состоянии покоя. Что удерживает его от соскальзывания вниз? Трение покоя обеспечивает сцепление бруска и доски.

Опыт 2. Прижмите свою руку к тетради, лежащей на столе, и передвиньте её. Тетрадь будет двигаться относительно стола, но находиться в покое относительно вашей ладони. С помощью чего вы принудили эту тетрадь двигаться? С помощью трения покоя тетради об руку. Трение покоя перемещает грузы, которые размещаются на подвижной ленте транспортёра, предотвращает развязывание шнурков, удерживает шурупы и гвозди в доске и т. п.

Если тело скользит по другому, то трение, возникающее при этом, называют трением скольжения. Такое трение возникает при движении саней или лыж по снегу, подошв обуви по земле.

Если одно тело катится по другому, то говорят о трении качения. При качении колес вагона, автомобиля, телеги, при перекатывании бочек по земле проявляется трение качения.

А от чего зависит сила трения ?

Опыт 3. Прикрепим к бруску динамометр и будем тянуть его, сообщая бруску равномерное движение (рис. 82).

Сила трения в физике - формулы и определения с примерами

При этом динамометр будет показывать силу, с которой мы тянем брусок, а тем самым и силу трения, возникающую во время движения бруска по поверхности стола. Положим на брусок грузики и повторим опыт. Динамометр зафиксирует большую силу трения.

Чем большая сила прижимает тело к поверхности, тем большая сила трения возникает при этом.

Выполним предыдущий опыт, но тело будем двигать по поверхности стекла, по бетону. Выясним, что сила трения зависит от материала и качества поверхности, по которой движется тело.

Сила трения зависит от материала и качества обработки поверхности, по которой движется тело.

Силу трения скольжения определяют по формуле:

Сила трения в физике - формулы и определения с примерами

где Сила трения в физике - формулы и определения с примерами — сила трения скольжения; N — сила реакции опоры, значение которой равно силе давления тела на поверхность скольжения; Сила трения в физике - формулы и определения с примерами — коэффициент трения скольжения. Если поверхность скольжения горизонтальна, то сила давления на неё равна весу тела, т. е. Сила трения в физике - формулы и определения с примерами, а Сила трения в физике - формулы и определения с примерами, где Сила трения в физике - формулы и определения с примерами ; Сила трения в физике - формулы и определения с примерами — масса тела.

В таблице 5 указаны коэффициенты трения скольжения для некоторых пар материалов.

Сила трения в физике - формулы и определения с примерами

Опыт. Положим деревянный брусок на круглые карандаши (рис. 83). Сила трения в физике - формулы и определения с примерами

Потянем брусок динамометром, карандаши за счёт трения между ними, бруском и доской начнут вращаться, а брусок — двигаться. Сила трения качения окажется меньше силы трения скольжения.

При одинаковых нагрузках сила трения качения всегда меньше силы трения скольжения.

Рассматривая швейную иглу, вы сразу заметите, что она отполирована до блеска. Для чего нужна такая полировка? А легко ли шить заржавевшей иглой? Здесь вы непосредственно ощущаете, какую роль играет трение в быту.

В природе и технике трение может быть и полезным, и вредным. Когда оно полезное, его стараются увеличить, а когда вредное — уменьшить.

Из-за трения изнашиваются механизмы и машины, стираются подошвы обуви и шины автомобилей, усложняется перемещение разных грузов. Но представьте, что трение исчезло. Тогда движущийся автомобиль не смог бы остановиться, а неподвижный — сдвинуться с места; пешеходы упали бы на дорогу и не смогли бы подняться; ткани распались бы на нити, так как они удерживаются трением; вы даже не смогли бы перелистать страницы этого учебника.

Вы, наверное, неоднократно замечали, что на автомобильных шинах есть рельефные рисунки (так называемые протекторы), которые размещены вдоль и поперёк шины (рис. 84).

Сила трения в физике - формулы и определения с примерамиОни сделаны для увеличения трения, т. е. силы сцепления колёс с полотном дороги. Поперечные полосы увеличивают сцепление колеса с полотном дороги, а продольные полосы и выступы, размещённые под углом, препятствуют боковому смещению, соскальзыванию автомобиля.

Во всех машинах вследствие трения нагреваются и изнашиваются подвижные части. Чтобы уменьшить трение, соприкасающиеся поверхности делают гладкими и между ними вводят смазочное масло, поскольку трение между поверхностью твердого тела и жидкостью значительно меньше, чем между поверхностями твёрдых тел. Вращающиеся валы машин и станков устанавливают на подшипниках. Подшипники качения бывают шариковыми и роликовыми (рис. 85). Они дают возможность уменьшить силу трения в 20—30 раз по сравнению с подшипниками скольжения.

Известно, что смазка трущихся поверхностей значительно уменьшает трение между ними. Почему же тяжелее удерживать топорище топора сухой рукой, чем влажной? Оказывается, что при смазке дерева мелкие волокна на его поверхности набухают, поэтому трение между рукой и топорищем увеличивается, что и помогает удерживать топор в руках.

Наблюдение. Когда вы стараетесь бежать в воде бассейна, реки или озера, то ощущаете большое сопротивление со стороны воды и не можете передвигаться быстро. Перенося лёгкие большие предметы в ветреную погоду, вы ощущаете такое сопротивление со стороны ветра, что вам очень тяжело идти. Когда в безветренную погоду вы стоите у дороги и мимо вас проезжает большой грузовой автомобиль на большой скорости, то вы обязательно ощутите ветер, сопровождающий движение автомобиля. Сила этого ветра тем больше, чем выше скорость автомобиля.

Силы трения, возникающие при движении тел в жидкости или газе, называют силами сопротивления среды.

Сила сопротивления зависит от формы тела. Ракетам, самолётам, подводным лодкам, кораблям и автомобилям придают обтекаемую форму, т. е. форму, при которой сила сопротивления минимальна.

Опыт. Возьмём два измерительных цилиндра, наполним один из них водой, а второй — постным или машинным маслом. Бросим одновременно в них одинаковые металлические шарики. В результате опыта увидим, что шарик в воде упадёт на дно быстрее, чем в масле, т. е. сила сопротивления движения шарика в масле больше, чем в воде.

Лодки, корабли не могут развить таких скоростей, какие развивают самолёты, так как сила сопротивления движения в воде намного больше, чем в воздухе.

Сила трения

Как наблюдать силу трения:

Взаимодействие тел, вследствие чего изменяются скорости этих тел, происходит не только при их столкновении. В природе можно наблюдать множество примеров, когда одно тело скользит или катится по поверхности другого. О взаимодействии этих тел можно судить по тому, что скорость этих тел изменяется. Скатившись с горы, камень даже на ровной поверхности со временем остановится. Хоккейная шайба двигается по льду в течение определенного времени, а потом останавливается.

Закрепим наклонно на столе доску, положим на нее шарик и отпустим. Шарик скатится, приобретя определенную скорость, прокатится по столу и, в конце концов, остановится. Если на стол положить стекло, то шарик прокатится на большее расстояние. Таким образом, причиной изменения скорости шарика является его взаимодействие со столом или стеклом.

В рассмотренных примерах скорости камня, шайбы, шарика уменьшались. Значит, на них действовала некоторая сила, направленная против движения. Эта сила возникла в результате взаимодействия тел, касающихся друг друга и осуществляющих взаимное перемещение. Движущийся камень взаимодействует с поверхностью Земли,  шайба — с поверхностью льда, шарик — с поверхностью стола или стекла. При движении тела в жидкости или газе тоже возникает сила трения.

Силу, возникающую при относительном перемещении соприкасающихся тел, называют силой трения.

Как измерить силу трения

Опыты показывают, что сила трения может иметь различные значения. Измерить ее можно при помощи динамометра. Положим деревянный брусок на доску, присоединим к нему крючок динамометра и начнем тянуть за него. Стрелка динамометра начнет отклоняться от нулевой отметки, а когда брусок начнет двигаться равномерно, остановится на определенном делении. Это и будет значение силы трения при движении бруска но поверхности доски. Сила трения всегда пропорциональна силе, с которой прижимается одно тело к другому. Эту зависимость можно выразить формулой:Сила трения в физике - формулы и определения с примерами

где Сила трения в физике - формулы и определения с примерами — сила трения; N — сила реакции опоры; Сила трения в физике - формулы и определения с примерами — коэффициент трения.

Коэффициент трения Сила трения в физике - формулы и определения с примерами зависит от качества поверхностей соприкасающихся тел и от веществ, с которых они изготовлены.

Сила трения в физике - формулы и определения с примерами

Почему возникает сила трения

Природу силы трения можно объяснить, если учесть свойства взаимодействующих тел. Поверхность каждого тела всегда имеет микроскопические неровности. При относительном перемещении двух тел эти неровности мешают взаимному смещению тел, что и проявляется как сила трения (рис. 53). Даже тщательная полировка не поможет преодолеть трение. Исследования показали, что трение даже будет возрастать. Так как в этом случае расстояния между молекулами тел уменьшаются, то можно сделать выводы, что трение связано с взаимодействием молекул.

Сила трения в физике - формулы и определения с примерами

Виды трения

Различают три вида трения: трение скольжения, трение качения и трение покоя.

Трение скольжения возникает тогда, когда одно тело скользит по поверхности другого. Трение качения возникает при качении одного тела шарообразной или цилиндрической формы по поверхности другого тела. Сила трения скольжения всегда больше силы трения качения. Этот факт хорошо известен грузчикам, которые вместо того, чтобы тянуть бочку, катят ее.

Как учитывают силы трения

Трение везде встречается в природе и может как содействовать, так и мешать деятельности человека. В каждом случае люди научились управлять этим явлением, создавая условия, когда силы трения уменьшаются или, наоборот, увеличиваются. Так, для увеличения безопасности движения автомобиля его шины изготавливают с шероховатой поверхностью, которая дополнительно имеет узорчатые углубления (рис. 54), что способствует увеличению силы трения колес об асфальт.

Сила трения в физике - формулы и определения с примерами

Во всех транспортных средствах есть тормоза, предназначенные для торможения, то есть для ускорения остановки автомобиля или поезда. Тормоза оснащены тормозными колодками, которые покрыты специальным материалом, коэффициент трения которого по стали велик (рис. 55).

Сила трения в физике - формулы и определения с примерамиСила трения в физике - формулы и определения с примерами

В то же время, бывают случаи, когда силу трения нужно существенно уменьшить. Тогда трущиеся поверхности разделяют жидкостью — минеральной смазкой или даже водой, как это происходит в стиральных машинах. Слой жидкости разделяет трущиеся поверхности, и они не взаимодействуют друг с другом (рис. 56).

На различных деталях современных машин и механизмов устанавливают шариковые или роликовые подшипники качения (рис. 57). Как правило, это две стальные обоймы, между которыми находятся металлические шарики или цилиндрики — ролики. Такие подшипники существенно уменьшают трение, так как в них действуют только силы трения качения, которые при равных условиях значительно меньше сил трения скольжения. Заполненные смазкой шариковые и роликовые подшипники обеспечивают быстрое, бесшумное и экономное вращение деталей.

Сила трения в физике - формулы и определения с примерами

Что такое сила трения

Трение, при котором твердые тела взаимодействуют своими поверхностями, называют внешним. Внутренним считают трение, возникающее во время движения жидкостей и газов.
 

Сила трения — это сила, возникающая в плоскости касания поверхностей двух тел, прижатых одно к другому, и препятствующая их относительному перемещению.

Сила трения возникает не только во время относительного движения тел, но и в случае их относительного покоя (сила трения покоя).

Сила трения покоя равна внешней силе, которая пытается сдвинуть тело с места. Она направлена противоположно направлению приложенной силы.

В зависимости от вида перемещения одного тела по другому различают трение скольжения и трение качения.

Сила трения скольжения прямо пропорциональна силе реакции опоры:Сила трения в физике - формулы и определения с примерами
где Сила трения в физике - формулы и определения с примерами — коэффициент пропорциональности, который называется коэффициентом трения скольжения и характеризует природу и качество соприкасающихся поверхностей (шероховатость, волнистость и т. д.). На коэффициент трения скольжения влияет наличие между трущимися поверхностями разных веществ (пленок, масел или смазок), их температура и т. д.
Сила трения в физике - формулы и определения с примерами

Если приложенная к телу сила Сила трения в физике - формулы и определения с примерами которая лежит в плоскости поверхности тел (рис. 2.21), недостаточна для того, чтобы вызвать скольжение данного тела относительно другого, то такая сила называется силой трения покоя (рис. 2.22, отрезок ОА).

Сила трения в физике - формулы и определения с примерами

Сила трения покоя во время взаимодействия изменяется от нуля до максимального значения (точка А). Когда сила F достигает этого значения, трение покоя переходит в
трение скольжения.

Тело начинает скользить. При этом сила трения скольжения несколько меньше силы трения покоя. Дальше сила трения скольжения уже остается постоянной.

При движении тела по наклонной плоскости (рис. 2.23) на силу реакции опоры влияет угол наклона этой плоскости к горизонту а: N = mg cos а.

Значения коэффициента трения скольжения в зависимости от характера трущихся поверхностей для сухого трения (без масел) приведены в таблице 1.

Сила трения качения имеет более сложную зависимость, также обусловленную деформацией соприкасающихся поверхностей.

Таблица 1

Коэффициент трения скольжения
Сила трения в физике - формулы и определения с примерами
 

Пример №1

По горизонтальной дороге тянут за веревку (под углом 30°) сани с грузом, общая масса которых 80 кг. Сила натяжения 50 Н. Определить коэффициент трения скольжения, если сани движутся с ускорением 0,15 Сила трения в физике - формулы и определения с примерами.
Дано:

m = 80 кг,

Сила трения в физике - формулы и определения с примерами = 30°,

F = 50 Н,

а = 0,15 Сила трения в физике - формулы и определения с примерами

Сила трения в физике - формулы и определения с примерами
Решение

На сани действуют силы: тяжести Сила трения в физике - формулы и определения с примерами

реакции дороги Сила трения в физике - формулы и определения с примерами натяжения Сила трения в физике - формулы и определения с примерами и трения Сила трения в физике - формулы и определения с примерами Выполним рисунок, связав систему координат с дорогой (рис. 2.24).

Сила трения в физике - формулы и определения с примерами

Сначала рассмотрим случай, когда сани движутся равномерно. Силу трения Сила трения в физике - формулы и определения с примерами уравновешивает проекция силы натяжения веревки на ось ОХ:

Сила трения в физике - формулы и определения с примерами

Тогда Сила трения в физике - формулы и определения с примерами
Для равноускоренного движения запишем второй закон механики Ньютона для саней в векторной форме:

Сила трения в физике - формулы и определения с примерами

В проекциях на оси координат данное уравнение будет иметь такой вид:

на ось ОХ: та = FcosСила трения в физике - формулы и определения с примерамиСила трения в физике - формулы и определения с примерами

на ось ОУ: 0 — FsinСила трения в физике - формулы и определения с примерами + N — mg.

Поскольку Сила трения в физике - формулы и определения с примерами составим систему уравнений:

Сила трения в физике - формулы и определения с примерами

Разделив первое уравнение на второе, получим
Сила трения в физике - формулы и определения с примерами
Ответ: Сила трения в физике - формулы и определения с примерами

Работа силы трения

Движение тел на Земле происходит под действием различных сил, но практически всегда присутствуют силы трения, силы сопротивления среды, в которой движется тело. Поэтому рассмотрим на частных примерах работу этих сил.

Проведем следующий опыт. Толкнем брусок, лежащий на столе. Он придет в движение, а затем остановится. В процессе движения на него действуют сила тяжести Сила трения в физике - формулы и определения с примерами, сила нормальной реакции стола Сила трения в физике - формулы и определения с примерами и сила трения скольженияСила трения в физике - формулы и определения с примерами (рис. 141). Под действием этих сил брусок движется. Равнодействующая сил:
Сила трения в физике - формулы и определения с примерами,    (1)

поскольку сила тяжести компенсируется силой нормальной реакции стола. По теореме об изменении кинетической энергии тела работа равнодействующей силы равна изменению кинетической энергии тела. Если в начальный момент времени скорость тела была равна Сила трения в физике - формулы и определения с примерами, а затем тело остановилось, то
Сила трения в физике - формулы и определения с примерами

C другой стороны, эта работа есть работа сил трения, т. е.:

Сила трения в физике - формулы и определения с примерами   (2)

Таким образом, работа силы трения скольжения отрицательна.

При скольжении одного тела по поверхности другого происходит, во-первых, деформация шероховатостей обеих поверхностей и, во-вторых, трущиеся тела нагреваются, т. е. повышается их температура. В этом можно легко убедиться, если потереть деревянный брусок о доску. Из курса физики 8-го класса известно, что температура тел определяется средней кинетической энергией движения молекул, из которых они состоят. Повышение температуры трущихся тел означает увеличение средней кинетической энергии хаотического движения молекул этих тел, а следовательно, их внутренней энергии. Таким образом, можно сказать, что начальная кинетическая энергия бруска превратилась во внутреннюю энергию трущихся тел.

Сила трения в физике - формулы и определения с примерами
Рис. 141

Работу силы трения скольжения мы можем легко подсчитать и иначе. По закону сухого трения Сила трения в физике - формулы и определения с примерами, где μ — коэффициент трения скольжения. Пусть брусок совершил перемещение Сила трения в физике - формулы и определения с примерами. Тогда работа силы трения будет равна:
Сила трения в физике - формулы и определения с примерами      (3)
так как Сила трения в физике - формулы и определения с примерами= 180o, cosСила трения в физике - формулы и определения с примерами = -l.

Из формулы (3) следует, что работа силы трения зависит от модуля перемещения тела. Если тело вернется в исходную точку, то работа силы трения не будет равна нулю. Таким образом, сила трения не является потенциальной силой. Для нее нельзя ввести понятие потенциальной энергии. Такие силы, работа которых зависит от формы траектории движения тела, называются непотенциальными или диссипативными (лат. dissipative — рассеяние).

Очевидно, что сила сопротивления среды (газа или жидкости) при движении некоторого тела, направленная в сторону, противоположную скорости тела, также совершает работу. Однако не надо думать, что работа сил трения всегда отрицательна. Ведь именно благодаря силе трения покоя человек и различные машины движутся по земле. Действительно, при ходьбе человек действует на поверхность Земли с силой Сила трения в физике - формулы и определения с примерами(кроме силы нормальной реакции), а по третьему закону Ньютона Земля действует на ногу человека с силой трения покоя Сила трения в физике - формулы и определения с примерами, равной по модулю Сила трения в физике - формулы и определения с примерами, но противоположно направленной (рис. 142). Благодаря этой силе человек и движется. Сила Сила трения в физике - формулы и определения с примерами направлена также, как и скорость человека, следовательно, и работа этой силы положительна.

Сила трения в физике - формулы и определения с примерами
Рис. 142

Если тело лежит на движущейся ленте транспортера, то именно благодаря силе трения оно приобретает скорость (рис. 143).

Сила трения в физике - формулы и определения с примерами
Рис. 143

Точно так же под действием силы трения покоя движутся и автомобили. На ведущие колеса автомобиля от мотора передается вращательный момент.

Колеса пытаются провернуться, следовательно, в горизонтальном направлении они действуют на поверхность земли с силой Сила трения в физике - формулы и определения с примерами (рис. 144). По третьему закону Ньютона земля действует на колеса, а следовательно, и на машину с силой трения покояСила трения в физике - формулы и определения с примерами. Если колесо проскальзывает, то с силой трения скольжения, направленной для ведущих колес в сторону движения автомобиля.

Сила трения в физике - формулы и определения с примерами
Рис. 144

Главные выводы

  1. Силы трения не являются потенциальными силами.
  2. Работа сил трения зависит от формы траектории движения тела. Работа сил трения по замкнутой траектории не равна нулю.
  3. Работа сил трения обычно отрицательна. Она идет на увеличение внутренней энергии взаимодействующих тел.

Сила трения и движение под действием силы трения

Сила трения возникает между соприкасающимися друг с другом телами и направлена вдоль поверхности соприкосновения против их относительного движения. Причиной возникновения силы трения являются неровности соприкасающихся поверхностей и «силы сцепления» (силы притяжения) между молекулами этих поверхностей. Возникновение таких сил между молекулами определяет электромагнитную природу силы трения.

Существуют три вида силы трения:

  • Сила трения скольжения — это сила трения, возникающая при скольжении одного тела по поверхности другого тела.
  • Сила трения качения — это сила трения, возникающая, когда одно тело катится по поверхности другого.
  • Сила трения покоя — это сила трения, возникающая между телами, находящимися в состоянии покоя друг относительно друга. Численно сила трения покоя равна силе (b) тяги, направленной параллельно поверхности соприкосновения неподвижных тел, и направлена против нее (b).

Сила трения в физике - формулы и определения с примерами

При определенном значении силы тяги тело начинает двигаться и скользить по поверхности другого тела — возникает сила трения скольжения.

Численное значение силы трения скольжения прямо пропорционально силе реакции опоры (силе давления) и равно максимальному значению силы трения покоя:

Сила трения в физике - формулы и определения с примерами

Где Сила трения в физике - формулы и определения с примерами — коэффициент пропорциональности, называемый коэффициентом трения скольжения. Коэффициент трения скольжения зависит от материалов, из которых изготовлены соприкасающиеся тела, и качества обработки соприкасающихся поверхностей. Сила трения в физике - формулы и определения с примерами — безразмерная величина, единица измерения отсутствует.

В зависимости от свойств соприкасающихся поверхностей силу трения называют сухой силой трения и силой сопротивления.

  • Сухое трение — это трение, возникающее между поверхностями соприкасающихся твердых тел.
  • Сила сопротивления — это сила, возникающая во время движения твердого тела в жидкости или газе.

Движение под действием силы трения

Исследуем разные движения тела массой Сила трения в физике - формулы и определения с примерами под действием силы трения:

Тело движется прямолинейно равномерно по горизонтальной поверхности

Все силы, действующие на тело, показаны на схеме (с). При равномерном движении тела его ускорение Сила трения в физике - формулы и определения с примерами следовательно, II закон Ньютона, или уравнение движения тела в векторном виде записывается так:

Сила трения в физике - формулы и определения с примерами

Выбрав координатную ось вдоль направления силы тяги (в направлении движения) и получив проекции всех сил на эту ось, можно написать уравнение движения (см: с):

Сила трения в физике - формулы и определения с примерами

Здесь было принято во внимание, что проекции силы реакции и силы тяжести на ось Сила трения в физике - формулы и определения с примерами равны нулю — эти векторы перпендикулярны оси.

Таким образом, модули сил, действующих на тело, движущееся равномерно прямолинейно по горизонтальной поверхности, попарно равны и компенсируют взаимное действие друг друга:

Сила трения в физике - формулы и определения с примерами

Тело движется прямолинейно равнопеременно по горизонтальной поверхности (d).

В этом случае уравнение движении тела в общем виде: 

Сила трения в физике - формулы и определения с примерами

Спроецировав силы на горизонтальную координатную ось, запишем уравнение движения в скалярном виде:

Сила трения в физике - формулы и определения с примерами

Любая величина, входящая в последнее выражение, с легкостью определяется.

На движущееся тело действует только сила трения

Так как сила трения всегда направлена против направления движения, то ускорение, сообщаемое этой силой, направлено против скорости движения тела. Поэтому, если на движущееся тело действует только сила трения, то оно тормозится. В этом случае уравнение движения записывается в виде:

Сила трения в физике - формулы и определения с примерами

Для ускорения тела имеем

Сила трения в физике - формулы и определения с примерами

Отсюда можно определить тормозной путь и время торможения тела, движущегося по горизонтальной дороге:

Сила трения в физике - формулы и определения с примерами

Тело движется по наклонной плоскости

Наклонная носкость — это плоскость, образующая определенный угол Сила трения в физике - формулы и определения с примерами с горизонтом. Как показано на рисунке, сила тяжести, действующая на тело, движущееся равномерно под действием силы тяги по наклонной плоскости, раскладывается на две составляющие силы: составляющую, параллельную поверхности Сила трения в физике - формулы и определения с примерами и составляющую, перпендикулярную поверхности Сила трения в физике - формулы и определения с примерами (е). В этом случае модуль силы реакции опоры равен модулю составляющей Сила трения в физике - формулы и определения с примерами

Сила трения в физике - формулы и определения с примерами

Сила трения в физике - формулы и определения с примерами

Уравнение движения тела по наклонной плоскости в общем виде записывается так:

Сила трения в физике - формулы и определения с примерами

Для решения уравнения выбираем прямоугольную систему координат XOY, находим проекции сил на ее оси и получаем систему двух уравнений:

Сила трения в физике - формулы и определения с примерами

Ввиду отсутствия движения вдоль оси OY Сила трения в физике - формулы и определения с примерами Учитывая этот факт и уравнения (2.39) — (2.41), можно определить ускорение тела:

Сила трения в физике - формулы и определения с примерами

Определение силы трения

При движении одного тела по поверхности другого (при попытке к такому движению) возникает сила трения, направленная против движения (против возможного движения).

Опыт показывает, что в земных условиях всякое неподдерживаемое механическое движение с течением времени прекращается под действием сил трения (сопротивления).

Трением называется взаимодействие между различными соприкасающимися телами, препятствующее их относительному перемещению.

Силы трения имеют электромагнитное происхождение, поскольку их появление обусловлено взаимодействием «пограничных» атомов, расположенных на поверхностях соприкасающихся тел. Вследствие этого, силы трения, как правило, действуют параллельно трущимся поверхностям.

Различают силы сухого трения (покоя, скольжения, качения) и вязкого трения (силы сопротивления, возникающие при движении в жидкости или газе).

Отметим, что действие сил трения приводит к переходу механической энергии во внутреннюю энергию тела.

Трение покоя

Силы трения покоя возникают между неподвижными телами при попытке сдвинуть одно из них (рис. 36).

Сила трения в физике - формулы и определения с примерами

Сила трения покоя равна по модулю и направлена противоположно силе, приложенной к телу, параллельно поверхности соприкасающихся тел. В зависимости от приложенной силы модуль силы трения Сила трения в физике - формулы и определения с примерами может меняться в пределах

Сила трения в физике - формулы и определения с примерами
Экспериментально установлено, что

Сила трения в физике - формулы и определения с примерами

где N — модуль силы нормальной реакции опоры в месте соприкосновения тел, Сила трения в физике - формулы и определения с примерами — коэффициент трения покоя, зависящий от свойств веществ соприкасающихся поверхностей и от степени их шероховатости (качества обработки). Установлено также, что коэффициент трения покоя не зависит от площади соприкасающихся поверхностей.

Согласно третьему закону Ньютона модуль силы нормальной реакции опоры N равен модулю силы нормального давления Сила трения в физике - формулы и определения с примерами
Трение скольжения. Сила трения скольжения Сила трения в физике - формулы и определения с примерами возникает между движущимися относительно друг друга телами и препятствует их относительному перемещению (рис. 37).

Сила трения в физике - формулы и определения с примерами

Она направлена противоположно скорости относительного движения поверхностей. Модуль силы трения скольжения Сила трения в физике - формулы и определения с примерами прямо пропорционален модулю силы нормальной реакции опоры и определяется по формуле

Сила трения в физике - формулы и определения с примерами

где Сила трения в физике - формулы и определения с примерами — коэффициент трения скольжения, зависящий от тех же параметров, что и коэффициент трения покоя Сила трения в физике - формулы и определения с примерами

Этот закон был установлен экспериментально и называется законом Кулона — Амонтона.

Точные измерения показывают, что коэффициент трения скольжения зависит также и от модуля скорости относительного движения соприкасающихся тел (при малых скоростях в большинстве случаев Сила трения в физике - формулы и определения с примерами
Как следует из формулы для модуля силы трения скольжения, коэффициент трения Сила трения в физике - формулы и определения с примерами можно выразить как отношение модулей силы трения к силе нормального давления:
Сила трения в физике - формулы и определения с примерами

Поверхность называется гладкой, если силы трения равны нулю при любом характере движения.

Вязкое трение

Эксперименты показывают, что при движении в жидкости или газе (сплошной среде) на тело действует сила вязкого трения Сила трения в физике - формулы и определения с примерами Она зависит от размеров и формы тела, от скорости его движения (рис. 38), а также свойств жидкости или газа. Сила вязкого трения Сила трения в физике - формулы и определения с примерами возникает также между слоями жидкости или газа при их относительном движении.

Сила трения в физике - формулы и определения с примерами

При небольших скоростях движения (малых по сравнению со скоростью звука в воздухе) можно считать, что модуль силы вязкого трения прямо пропорционален скорости движения тела:
Сила трения в физике - формулы и определения с примерами

а при больших скоростях — квадрату скорости:

Сила трения в физике - формулы и определения с примерами

где Сила трения в физике - формулы и определения с примерами — постоянные коэффициенты сопротивления (рис. 39).

Сила трения в физике - формулы и определения с примерами

  • Заказать решение задач по физике

Откуда появилась сила трения

Сила трения в физике - формулы и определения с примерами

Почему профили самолетов и подводных лодок напоминают контуры тела дельфина? Почему зимой автомобили «переобувают» в шипованную резину? Почему трудно двигаться в гололед? Как «падает» парашютист? Как уменьшить силу трения? А может, ее не стоит уменьшать, а наоборот, нужно увеличивать? Что будет, если трение исчезнет вообще?

При любом движении тело обязательно контактирует с микро- или макротелами вокруг (поверхностью другого тела, частицами жидкости или газа, внутри которых тело движется, и т. д.). При таком контакте возникают силы, замедляющие движение тела, — силы трения.

Сила трения Сила трения в физике - формулы и определения с примерами — это сила, возникающая при движении или попытке движения одного тела по поверхности другого либо при движении тела внутри жидкой или газообразной среды.

Сила трения всегда направлена вдоль поверхности соприкасающихся тел и противоположно направлению скорости их относительного движения (рис. 13.1).

Сила трения в физике - формулы и определения с примерами

Рис. 13.1. Относительно поверхности снега и относительно воздуха лыжник движется вправо, поэтому сила трения Сила трения в физике - формулы и определения с примерами и сила сопротивления Сила трения в физике - формулы и определения с примерами, действующие на лыжника, направлены влево. Снег относительно лыжника движется влево, со стороны лыжника на снег действует сила трения Сила трения в физике - формулы и определения с примерами, направленная вправо

Трение между поверхностью твердого тела и окружающей жидкой или газообразной средой называют сопротивлением среды или жидким (вязким) трением. Трение между поверхностями двух соприкасающихся твердых тел называют сухим трением.

Почему возникает сила сухого трения

Если рассмотреть поверхность любого тела в лупу, можно увидеть множество мелких неровностей. Когда одно тело скользит или пытается скользить по поверхности другого, неровности цепляются друг за друга и деформируются. Возникают силы упругости, направленные в сторону, противоположную деформации (рис. 13.2).

Сила трения в физике - формулы и определения с примерами

Рис. 13.2. Один из механизмов возникновения сухого трения связан с наличием неровностей на поверхностях соприкасающихся тел

Это одна из причин возникновения силы сухого трения. Есть и другие причины. Так, в некоторых местах выступы тел плотно прижаты друг к другу — расстояние между ними настолько мало, что действуют силы межмолекулярного притяжения, в результате чего выступы оказываются как бы «склеенными». Понятно, что такое «склеивание» происходит в ходе всего движения и препятствует ему.

И сила упругости, и сила межмолекулярного притяжения имеют электромагнитное происхождение, поэтому природа силы сухого трения — электромагнитная.

Какие существуют виды сухого трения

Различают три вида сухого трения: трение покоя, трение скольжения, трение качения. Если вы попробуете, прикладывая небольшую силу, сдвинуть с места санки с тяжелым грузом, они не сдвинутся, поскольку возникнет сила трения покоя, которая уравновесит прилагаемую внешнюю силу.

Сила трения покоя Сила трения в физике - формулы и определения с примерами — это сила трения, возникающая между соприкасающимися поверхностями двух тел и препятствующая возникновению их относительного движения.

Сила трения в физике - формулы и определения с примерами

Рис. 13.4. Внешние силы пытаются сдвинуть тело. Сила трения покоя, возникающая при этом, уравновешивает внешние силы, и тело находится в состоянии покоя

Сила трения покоя всегда равна по модулю и противоположна по направлению равнодействующей внешних сил Сила трения в физике - формулы и определения с примерами, которая пытается сдвинуть тело с места (рис. 13.4):

Сила трения в физике - формулы и определения с примерами

Чем большая сила будет приложена, тем больше будет сила трения покоя. Наконец при определенном значении равнодействующей внешних сил (а следовательно, и силы трения покоя) тело сдвинется с места. То есть сила трения покоя имеет некоторое максимальное значение.

Чаще всего действие силы трения покоя «полезно»: благодаря ей вещи не выскальзывают из рук, грифель карандаша оставляет след на бумаге; эта сила позволяет выполнять повороты, удерживает корни растений в почве. Благодаря силе трения покоя передвигаются люди, животные, транспорт (рис. 13.5).

Сила трения в физике - формулы и определения с примерами

Рис. 13.5. Шины автомобиля в момент соприкосновения с поверхностью дороги по сути пытаются осуществить движение назад. В результате возникает сила трения покоя, направленная вперед, — движущая сила

В технике, на транспорте, в быту часто принимают меры для увеличения максимальной силы трения покоя: на ступеньки и обувь наклеивают противоскользящие накладки, автомобили «переобувают» в зимние шины и т. д.

После того как равнодействующая внешних сил становится равной максимальной силе трения покоя, тело начинает скольжение, — и тогда говорят о силе трения скольжения.

Сила трения скольжения Сила трения в физике - формулы и определения с примерами — это сила, возникающая при скольжении одного тела по поверхности другого и направленная в сторону, противоположную направлению относительной скорости движения тел.

Сила трения скольжения действует вдоль поверхности соприкосновения тел, и она немного меньше максимальной силы трения покоя (рис. 13.6).

Сила трения в физике - формулы и определения с примерами

Рис. 13.6. Когда сила трения покоя достигает максимального значения, тело трогается с места (начинает скольжение)

Именно поэтому тела сдвигаются с места рывком и сдвинуть их труднее, чем затем перемещать. Это особенно заметно, когда тела массивные. Ваш жизненный опыт показывает, что сила трения скольжения зависит от свойств соприкасающихся поверхностей и увеличивается с увеличением силы нормальной реакции опоры (рис. 13.7). Закон, отражающий зависимость Сила трения в физике - формулы и определения с примерами был экспериментально установлен французским ученым Г. Амонтоном (1663–1705) и проверен его соотечественником Ш. Кулоном (1736–1806), поэтому называется закон Амонтона — Кулона:

Сила трения скольжения не зависит от площади соприкосновения тел и прямо пропорциональна силе N нормальной реакции опоры:

Сила трения в физике - формулы и определения с примерами

Здесь Сила трения в физике - формулы и определения с примерамикоэффициент трения скольжения, который зависит от материалов и качества обработки соприкасающихся поверхностей, незначительно зависит от относительной скорости движения соприкасающихся поверхностей и является безразмерной величиной:

Сила трения в физике - формулы и определения с примерами

Сила трения в физике - формулы и определения с примерами

Рис. 13.7. Сила трения скольжения зависит от качества и рода поверхностей (а) и увеличивается с увеличением силы нормальной реакции опоры (б)

Значения коэффициентов трения скольжения устанавливают исключительно экспериментально. Обычно таблицы коэффициентов трения скольжения содержат ориентировочные средние значения для пар материалов (см. таблицу).

Материалы Коэффициент трения скольжения
Сталь по льду 0,02
Сталь по стали м
Бронза по бронзе 0,20
Дерево по дереву 0,25
Бумага (картон) по дереву 0,40
Резина по бетону 0,75

Силу трения скольжения можно уменьшить, смазав соприкасающиеся поверхности. Твердая смазка изменяет качество поверхности; жидкая смазка отдаляет соприкасающиеся поверхности друг от друга — сухое трение заменяется значительно более слабым жидким трением.

Трение существенно уменьшится, если между соприкасающимися поверхностями расположить твердые катки, то есть скольжение заменить качением. Опыты показывают, что при одинаковых условиях сила трения качения в десятки раз меньше, чем сила трения скольжения.

Одна из причин возникновения силы трения качения заключается в том, что поверхность, по которой движется шарообразное тело (цилиндр, колесо, шар), деформируется, поэтому тело все время словно закатывается на небольшую наклонную плоскость (рис. 13.8).

Сила трения в физике - формулы и определения с примерами

Чем больше деформация поверхности, тем больше угол наклона плоскости и тем больше сила трения качения. Именно поэтому сила трения качения:

  • уменьшается с увеличением твердости поверхности, по которой катится тело, и твердости материала, из которого изготовлено тело;
  • увеличивается с увеличением давления тела на поверхность;
  • уменьшается с увеличением радиуса тела.

Сила сопротивления среды

Сила сопротивления среды (сила вязкого трения) Сила трения в физике - формулы и определения с примерами — сила, возникающая при движении тела внутри жидкости или газа.

Сила трения в физике - формулы и определения с примерами

Рассмотрим причины возникновения силы сопротивления среды.

  1. Ламинарное обтекание. Если твердое тело движется внутри жидкости или газа, то прилегающие слои среды движутся вместе с телом (рис. 13.9). Чем больше вязкость среды, тем больше ее слоев вовлекаются в движение.
  2. Лобовое сопротивление. Частицы среды сталкиваются с телом и замедляют его движение.
  3. Вихревое обтекание. Если тело движется с большой скоростью, то ламинарное обтекание переходит в вихревое: непосредственно за телом образуется зона пониженного давления, и тело как бы втягивается в эту зону, замедляя свое движение.

Сила трения в физике - формулы и определения с примерами

Сила сопротивления среды существенно зависит от формы тела (рис. 13.10).

Сила сопротивления среды увеличивается:

Сила трения в физике - формулы и определения с примерами

Обратите внимание! Не существует силы жидкого трения покоя. То есть если тело, расположенное в жидкой или газообразной среде, находится в состоянии покоя относительно среды, то сила сопротивления среды на него не действует.

Пример №2

На горизонтальной дороге автомобиль выполняет поворот радиусом 45 м. Какую наибольшую скорость может иметь автомобиль, чтобы «вписаться» в поворот, если коэффициент трения скольжения шин об асфальт Сила трения в физике - формулы и определения с примерами= 0,5?

Анализ физической проблемы. Автомобиль «не впишется» в поворот, если Сила трения в физике - формулы и определения с примерами, направленная к центру окружности, достигнет максимального значения и «перейдет» в силу трения скольжения. Обратите внимание: кроме силы трения покоя, направленной к центру окружности и препятствующей боковому скольжению автомобиля, существует еще сила трения покоя, препятствующая проскальзыванию колес вдоль направления движения автомобиля, — она и является силой тяги автомобиля (рис. 13.12).

Сила трения в физике - формулы и определения с примерами

Выполним пояснительный рисунок, указав силы, действующие на автомобиль, и направление ускорения его движения. Систему координат свяжем с телом на поверхности Земли.

Сила трения в физике - формулы и определения с примерамиСила трения в физике - формулы и определения с примерами

Решение:

Запишем второй закон Ньютона:

Сила трения в физике - формулы и определения с примерами

Спроецируем уравнения на оси координат:

Сила трения в физике - формулы и определения с примерами

Поскольку Сила трения в физике - формулы и определения с примерами, то получим: Сила трения в физике - формулы и определения с примерами. Проверим единицу, найдем значение искомой величины:

Сила трения в физике - формулы и определения с примерами

Ответ: Сила трения в физике - формулы и определения с примерами = 15 м/с.

Выводы:

  • Сила трения — это сила, возникающая при движении или попытке движения одного тела по поверхности другого, а также при движении тела внутри жидкой или газообразной среды. Сила трения всегда направлена вдоль поверхностей соприкасающихся тел и противоположно скорости их относительного движения.
  • Различают силы трения покоя, трения скольжения, трения качения и сопротивления среды. Все эти силы, кроме силы трения качения, имеют электромагнитную природу.
  • Сила трения покоя равна по модулю и противоположна по направлению равнодействующей внешних сил, действующих на тело: Сила трения в физике - формулы и определения с примерами
  • Сила трения скольжения прямо пропорциональна силе нормальной реакции опоры: Сила трения в физике - формулы и определения с примерами. Коэффициент трения скольжения µ зависит от материалов соприкасающихся поверхностей и качества их обработки.
  • Сила трения качения прямо пропорциональна силе нормальной реакции опоры, намного меньше силы трения скольжения, зависит от радиуса тела, материала и твердости соприкасающихся поверхностей.
  • Сила сопротивления среды существенно зависит от формы тела, увеличивается с увеличением скорости движения тела, площади его поперечного сечения, а также с увеличением вязкости и плотности среды.

Вычисление силы трения

Французский физик Гийом Амонтон (1663–1705), размышляя о роли трения, писал: «Всем нам случалось выходить в гололедицу: сколько усилий стоило нам, чтобы удерживаться от падения, сколько смешных движений приходилось нам проделывать, чтобы устоять!.. Представим, что трение исчезло вовсе. Тогда никакие тела, будь они величиной с каменную глыбу или малы, как песчинки, никогда не удержатся одно на другом: все будет скользить и катиться…. Не будь трения, Земля представляла бы собой шар без неровностей, подобный жидкой капле».

Сила трения покоя

Если вы пытаетесь передвинуть тяжелое тело, например большой ящик, и не можете сдвинуть его с места, это означает, что силу, которую вы прикладываете к ящику, уравновешивает сила трения покоя, возникающая между полом и нижней поверхностью ящика (рис. 21.1).

Сила трения в физике - формулы и определения с примерами

Сила трения покоя Сила трения в физике - формулы и определения с примерами — это сила, возникающая между двумя соприкасающимися телами при попытке сдвинуть одно тело относительно другого и направленная в сторону, противоположную той, в которую двигалось бы тело, если бы трения не было.

Сила трения покоя приложена вдоль поверхности, которой тело соприкасается с другим телом, и по значению равна силе Сила трения в физике - формулы и определения с примерами пытающейся сдвинуть тело с места (рис. 21.2): Сила трения в физике - формулы и определения с примерами

Сила трения в физике - формулы и определения с примерами

При увеличении силы Сила трения в физике - формулы и определения с примерами которая пытается сдвинуть тело, увеличивается и сила трения покоя. Когда сила Сила трения в физике - формулы и определения с примерами достигает такого значения, что тело вот-вот начнет движение, сила трения покоя максимальна. С началом движения сила трения покоя переходит в силу трения скольжения. Таким образом, для каждого случая сила трения покоя не может превышать некоторого максимального значения. Чаще всего действие силы трения покоя очень «полезно»: благодаря ей предметы не выскальзывают из рук, не развязываются узлы; эта сила удерживает песчинки в куче песка, тяжелые камни на склоне горы, корни растений в почве. Именно сила трения покоя является той силой, благодаря которой движутся люди, животные, транспорт (рис. 21.3).

Сила трения в физике - формулы и определения с примерами

В технике, на транспорте, в быту часто принимают меры, чтобы поверхность одного тела не двигалась относительно поверхности другого. Например, для увеличения максимальной силы трения покоя тротуары во время гололедицы посыпают песком, зимой автомобили «переобувают» в зимние шины. Попробуйте привести еще несколько подобных примеров.

От чего зависит сила трения скольжения

Сила трения в физике - формулы и определения с примерами — это сила, которая возникает при скольжении одного тела по поверхности другого и направлена в сторону, противоположную направлению движения тела. Сила трения скольжения действует вдоль поверхности соприкосновения тел (рис. 21.4) и немного меньше максимальной силы трения покоя.

Сила трения в физике - формулы и определения с примерами

Именно поэтому тела сдвигаются с места рывком и сдвинуть их тяжелее, чем затем двигать. Это особенно заметно, когда тела массивные. Прикрепим к деревянному бруску крючок динамометра и будем равномерно тянуть брусок по горизонтальной поверхности (рис. 21.5). На брусок в направлении его движения действует сила упругости со стороны пружины динамометра, а в противоположном направлении — сила трения скольжения. Брусок движется равномерно, поэтому сила упругости уравновешивает силу трения скольжения. Следовательно, динамометр показывает значение силы трения скольжения. Рассмотрите рис. 21.5 и сделайте вывод о том, как зависит сила трения скольжения от свойств соприкасающихся поверхностей. Обратите внимание: если провести те же опыты, перевернув брусок на меньшую грань, показания динамометра не изменятся.

Сила трения в физике - формулы и определения с примерами

Сила трения скольжения не зависит от площади соприкасающихся поверхностей. Проведем еще один опыт. Положим на брусок дополнительный груз, увеличив таким образом силу нормальной реакции опоры (рис. 21.6). Опыт покажет, что сила трения скольжения возросла.

Сила трения в физике - формулы и определения с примерами

Сила трения скольжения прямо пропорциональна силе нормальной реакции опоры*: Сила трения в физике - формулы и определения с примерами где N — сила нормальной реакции опоры; µ — коэффициент пропорциональности, который называют коэффициент трения скольжения.

Этот закон был установлен французским ученым Г. Амонтоном и проверен его соотечественником Ш. Кулоном, поэтому получил название закон Амонтона — Кулона.

Поскольку и силу трения скольжения, и силу нормальной реакции опоры измеряют в ньютонах, коэффициент трения скольжения — величина, не имеющая размерности: Сила трения в физике - формулы и определения с примерами Коэффициент трения скольжения определяется, в частности, материалами, из которых изготовлены соприкасающиеся тела, и качеством обработки их поверхностей. Значения коэффициентов трения скольжения устанавливают экспериментально. Таблицы коэффициентов трения скольжения обычно содержат ориентировочные средние значения для пар материалов (см. таблицу).

Сила трения в физике - формулы и определения с примерами

Причины возникновения и способы уменьшения силы трения

Поверхности твердых тел всегда шероховатые, неровные. При движении или попытке движения неровности цепляются друг за друга и деформируются или даже сминаются. В результате возникает сила, противодействующая движению тела, — сила трения (рис. 21.7).

Сила трения в физике - формулы и определения с примерами

Сила трения, как и сила упругости, — проявление сил межмолекулярного взаимодействия. Казалось бы, для уменьшения силы трения нужно тщательно отполировать поверхности и таким образом свести неровности к минимуму. Однако в таком случае поверхности будут настолько плотно прилегать друг к другу, что значительное количество молекул окажется на расстоянии, на котором становится существенным межмолекулярное притяжение. В результате сила трения возрастет*. Силу трения скольжения можно уменьшить, смазав соприкасающиеся поверхности. Смазка (как правило, жидкая), попав между соприкасающимися поверхностями, отдалит их друг от друга. То есть будут скользить не поверхности тел, а слои смазки, — трение скольжения (так называемое сухое трение) сменится вязким (жидким) трением, при котором сила трения значительно меньше.

Исследование трения и обоснование причин его возникновения достаточно сложны и вы ходят за рамки школьного курса физики.

Сила трения качения

Давний опыт человечества показывает, что, например, каменную глыбу легче перекатить на бревнах, чем просто тащить по земле. Если одно тело катится вдоль поверхности другого, то мы имеем дело с трением качения. Сила трения качения обычно намного меньше, чем сила трения скольжения (рис. 21.8, 21.9).

Сила трения в физике - формулы и определения с примерами

Сила трения в физике - формулы и определения с примерами

Поэтому для уменьшения силы трения люди издавна используют колесо, а в различных механизмах — подшипники (рис. 21.10).

Сила трения в физике - формулы и определения с примерами

Пример №3

Чтобы равномерно двигать по столу книгу массой 1 кг, нужно приложить горизонтальную силу 2 Н. Чему равен коэффициент трения скольжения между книгой и столом? Анализ физической проблемы. Выполним пояснительный рисунок, на котором изобразим все силы, действующие на книгу: Сила трения в физике - формулы и определения с примерами — сила тяжести; Сила трения в физике - формулы и определения с примерами — сила нормальной реакции опоры; Сила трения в физике - формулы и определения с примерами — сила, под действием которой книга движется по поверхности стола; Сила трения в физике - формулы и определения с примерами — сила трения скольжения. Книга движется равномерно, следовательно, силы, действующие на нее, попарно скомпенсированы: Сила трения в физике - формулы и определения с примерами Исходя из этого и найдем искомый коэффициент трения.

Сила трения в физике - формулы и определения с примерами

Дано:

Сила трения в физике - формулы и определения с примерами,Сила трения в физике - формулы и определения с примерами,Сила трения в физике - формулы и определения с примерами

Найти:

Сила трения в физике - формулы и определения с примерами

Решение:

По формуле для определения силы трения скольжения имеем:

Сила трения в физике - формулы и определения с примерами

Поскольку Сила трения в физике - формулы и определения с примерами а Сила трения в физике - формулы и определения с примерами

Проверим единицу, найдем значение искомой величины:

Сила трения в физике - формулы и определения с примерами

Анализ результатов: коэффициент трения 0,2 соответствует паре «дерево по дереву»; результат правдоподобен. Ответ: µ=0,2.

Итоги:

Сила трения покоя Сила трения в физике - формулы и определения с примерами — это сила, возникающая между двумя соприкасающимися телами при попытке сдвинуть одно тело относительно другого. Сила трения покоя всегда препятствует появлению относительного движения соприкасающихся тел; она равна по значению и противоположна по направлению силе F, пытающейся сдвинуть тело с места: Сила трения в физике - формулы и определения с примерами

Сила трения скольжения Сила трения в физике - формулы и определения с примерами — это сила, возникающая при скольжении одного тела по поверхности другого. Сила трения скольжения прямо пропорциональна силе нормальной реакции опоры Сила трения в физике - формулы и определения с примерами, где µ — коэффициент трения скольжения, зависящий от материалов, из которых изготовлены соприкасающиеся тела, качества обработки их поверхностей. Смазка поверхностей существенно уменьшает силу трения. При качении одного тела по поверхности другого возникает сила трения качения, которая обычно меньше силы трения скольжения.

  • Вес тела в физике
  • Закон всемирного тяготения
  • Свободное падение тела
  • Равнодействующая сила и движение тела под действием нескольких сил 
  • Сила тяжести в физике
  • Сила упругости в физике и закон Гука
  • Деформация в физике
  • Плотность вещества в физике

Уравнение силы трения на наклонной плоскости

Задачи по динамике.

I и II закон Ньютона.

Ввод и направление осей.

Проецирование сил на оси.

Решение систем уравнений.

Самые типовые задачи по динамике

Начнем с I и II законов Ньютона.

Откроем учебник физики и прочтем. I закон Ньютона: существуют такие инерциальные системы отсчета в которых. Закроем такой учебник, я тоже не понимаю. Ладно шучу, понимаю, но объясню проще.

I закон Ньютона: если тело стоит на месте либо движется равномерно (без ускорения), сумма действующих на него сил равна нулю.

Вывод: Если тело движется с постоянной скоростью или стоит на месте векторная сумма сил будет ноль.

II закон Ньютона: если тело движется равноускоренно или равнозамедленно (с ускорением), сумма сил, действующих на него, равна произведению массы на ускорение.

Вывод: Если тело двигается с изменяющейся скоростью, то векторная сумма сил, которые как-то влияют на это тело ( сила тяги, сила трения, сила сопротивления воздуха), равна массе этого тело умножить на ускорение.

При этом одно и то же тело чаще всего движется по-разному (равномерно или с ускорением) в разных осях. Рассмотрим именно такой пример.

Задача 1. Определите коэффициент трения шин автомобиля массой 600 кг, если сила тяги двигателя 4500 Н вызывает ускорение 5 м/с².

Обязательно в таких задачах делать рисунок, и показывать силы, которые дествуют на машину:

На Ось Х: движение с ускорением

На Ось Y: нет движения (здесь координата, как была ноль так и останется, машина не поднимает в горы или спускается вниз)

Те силы, направление которых совпадает с направлением осей, будут с плюсом, в противоположном случае — с минусом.

По оси X: сила тяги направлена вправо, так же как и ось X, ускорение так же направлено вправо.

Fтр = μN, где N — сила реакции опоры. На оси Y: N = mg, тогда в данной задаче Fтр = μmg.

Коэффициент трения — безразмерная величина. Следовательно, единиц измерения нет.

Задача 2. Груз массой 5кг, привязанный к невесомой нерастяжимой нити, поднимают вверх с ускорением 3м/с². Определите силу натяжения нити.

Сделаем рисунок, покажем силы, которые дествуют на груз

T — сила натяжения нити

На ось X: нет сил

Разберемся с направлением сил на ось Y:

Выразим T (силу натяжения) и подставим числительные значения:

Самое главное не запутаться с направлением сил (по оси или против), все остальное сделает калькулятор или всеми любимый столбик.

Далеко не всегда все силы, действующие на тело, направлены вдоль осей.

Простой пример: мальчик тянет санки

Если мы так же построим оси X и Y, то сила натяжения (тяги) не будет лежать ни на одной из осей.

Чтобы спроецировать силу тяги на оси, вспомним прямоугольный треугольник.

Отношение противолежащего катета к гипотенузе — это синус.

Отношение прилежащего катета к гипотенузе — это косинус.

Сила тяги на ось Y — отрезок (вектор) BC.

Сила тяги на ось X — отрезок (вектор) AC.

Если это непонятно, посмотрите задачу №4.

Чем длинее будет верека и, соответсвенно, меньше угол α, тем проще будет тянуть санки. Идеальный вариант, когда веревка параллельна земле , ведь сила, которая действуют на ось X— это Fнcosα. При каком угле косинус максимален? Чем больше будет этот катет, тем сильнее горизонтальная сила.

Задача 3. Брусок подвешен на двух нитях. Сила натяжения первой составляет 34 Н, второй — 21Н, θ1 = 45°, θ2 = 60°. Найдите массу бруска.

Введем оси и спроецируем силы:

Получаем два прямоугольных треугольника. Гипотенузы AB и KL — силы натяжения. LM и BC — проекции на ось X, AC и KM — на ось Y.

Задача 4. Брусок массой 5 кг (масса в этой задаче не нужна, но, чтобы в уравнениях все было известно, возьмем конкретное значение) соскальзывает с плоскости, которая наклонена под углом 45°, с коэффициентом трения μ = 0,1. Найдите ускорение движения бруска?

Когда же есть наклонная плоскость, оси (X и Y) лучше всего направить по направлению движения тела. Некоторые силы в данном случае ( здесь это mg) не будут лежать ни на одной из осей. Эту силу нужно спроецировать, чтобы она имела такое же направление, как и взятые оси.
Всегда ΔABC подобен ΔKOM в таких задачах (по прямому углу и углу наклона плоскости).

Рассмотрим поподробнее ΔKOM:

Получим, что KO лежит на оси Y, и проекция mg на ось Y будет с косинусом. А вектор MK коллинеарен (параллелен) оси X, проекция mg на ось X будет с синусом, и вектор МК направлен против оси X (то есть будет с минусом).

Не забываем, что, если направления оси и силы не совпадают, ее нужно взять с минусом!

Из оси Y выражаем N и подставляем в уравнение оси X, находим ускорение:

Как видно, массу в числителе можно вынести за скобки и сократить со знаменаталем. Тогда знать ее не обязательно, получить ответ реально и без нее.
Да-да, в идеальных условиях (когда нет силы сопротивления воздуха и т.п.), что перо, что гиря скатятся (упадут) за одно и тоже время.

Задача 5. Автобус съезжает с горки под уклоном 60° с ускорением 8 м/с² и с силой тяги 8 кН. Коэффициент трения шин об асфальт равен 0,4. Найдите массу автобуса.

Сделаем рисунок с силами:

Введем оси X и Y. Спроецируем mg на оси:

Запишем второй закон Ньютона на X и Y:

Задача 6. Поезд движется по закруглению радиуса 800 м со скоростью 72 км/ч. Определить, на сколько внешний рельс должен быть выше внутреннего. Расстояние между рельсами 1,5 м.

Самое сложное — понять, какие силы куда действуют, и как угол влияет на них.

Вспомни, когда едешь по кругу на машине или в автобусе, куда тебя выталкивает? Для этого и нужен наклон, чтобы поезд не упал набок!

Угол α задает отношение разницы высоты рельсов к расстоянию между ними (если бы рельсы находились горизонтально)

Запишем какие силы действуют на оси:

Ускорение в данной задачи центростремительное!

Поделим одно уравнение на другое:

Тангенс — это отношение противолежащего катета к прилежащему:

Как мы выяснили, решение подобных задач сводится к расстановке направлений сил, проецированию их на оси и к решению систем уравнений, почти сущий пустяк.

В качестве закрепления материала решите несколько похожих задач с подсказками и ответами.

Глава 6. Запрягаемся в упряжку: наклонные плоскости и трение

  • Постигаем гравитацию
  • Изучаем влияние наклона плоскости
  • Учитываем силы трения
  • Измеряем дальность полета под действием силы тяжести

Сила гравитационного притяжения — вот основная тема этой главы. В главе 5 было показано, что для ее преодоления требуется применять силу. В этой главе будет представлены способы влияния гравитационного притяжения и трения на движение объектов по наклонным плоскостям. Кроме того, будет показано, как гравитация влияет на траекторию полета объекта.

Разбираемся с гравитацией

На поверхности Земли сила гравитационного притяжения ​ ( mathbf ) ​ (или сила тяжести) постоянна и равна ​ ( mmathbf ) ​, где ​ ( m ) ​ — это масса объекта, a ​ ( mathbf ) ​ — ускорение свободного падения под действием силы тяжести, равное 9,8 м/с 2 .

Ускорение — это вектор, а значит, он имеет величину, направление и точку приложения (подробнее об этом см. главу 4). Уравнение ( mathbf=mmathbf ) интересно тем, что ускорение свободного падения объекта ​ ( g ) ​ не зависит от массы объекта.

Поскольку ускорение свободного падения не зависит от массы объекта, то более тяжелый объект падает нисколько не быстрее, чем более легкий объект. Сила тяжести сообщает свободно падающим телам одинаковое направленное вниз ускорение ( mathbf ) (на поверхности Земли равное ( mathbf ) ), независимо от их массы.

Сказанное выше относится к объектам вблизи поверхности Земли, а в главе 7 рассматриваются другие ситуации вдали от Земли (например, на орбите Луны), где сила тяжести и ускорение свободного падения имеют другие значения. Чем дальше вы находитесь от центра Земли, тем меньше сила тяжести и ускорение свободного падения. В примерах этой главы ускорение свободного падения направлено вниз. Но это не значит, что оно влияет только на движение предметов вертикально вниз. Здесь рассматриваются также примеры движения объектов под углом к вертикали.

Движемся по наклонной плоскости

В курсе физики часто упоминаются наклонные плоскости и рассматривается движение объектов по ним. Взгляните на рис. 6.1. На нем показана тележка, которая скатывается по наклонной плоскости. Тележка движется не строго вертикально, а вдоль плоскости, наклоненной под углом ​ ( theta ) ​ к горизонтали.

Допустим, что угол ( theta ) = 30°, а длина наклонной плоскости равна 5 метрам. До какой скорости разгонится тележка в конце наклонной плоскости? Сила тяжести сообщит тележке ускорение, но учтите, что вдоль наклонной плоскости ускорение будет отличаться от ускорения свободного падения. Дело в том, что разгон вдоль наклонной плоскости будет выполнять только компонента силы тяжести вдоль этой наклонной плоскости.

Чему равна компонента силы тяжести, действующей вдоль наклонной плоскости, если на тележку действует направленная вертикально сила тяжести ( mathbf ) ? Взгляните на рис. 6.2, на котором показаны упомянутые выше угол ( theta ) и вектор силы ( mathbf ) (подробнее о векторах см. главу 4). Для определения компоненты силы тяжести, действующей вдоль наклонной плоскости, нужно определить угол между вектором силы ( mathbf ) и наклонной плоскостью. Для этого потребуются элементарные сведения из геометрии (подробности см. в главе 2), а именно то, что сумма углов треугольника равна 180°. Угол между вектором силы ( mathbf ) и основанием наклонной плоскости равен 90°, а угол между наклонной плоскостью и ее основанием равен ( theta ) . Поэтому, глядя на рис. 6.2 , можно легко определить угол между вектором силы ( mathbf ) и наклонной плоскостью: 180°-90°- ( theta ) или 90°- ( theta ) .

Вычисляем углы

Преподаватели физики используют особый способ вычисления углов между векторами и наклонными плоскостями. Однако читателям книги можно раскрыть этот “секрет” определения угла ( theta ) . Для начала обратите внимание на то, что если ( theta ) стремится к 0°, то угол между вектором силы ( mathbf ) и наклонной плоскостью стремится к 90°. И наоборот, если ( theta ) стремится к 90°, то угол между вектором силы ( mathbf ) и наклонной плоскостью стремится к 0°. На основании этого простого наблюдения можно предположить, что угол между вектором силы ( mathbf ) и наклонной плоскостью равняется 90°- ( theta ) . Как видите, для определения взаимосвязи между углами бывает полезно попробовать поменять значения некоторых углов от 0° до 90°.

Ищем компоненту вектора силы Fg вдоль наклонной плоскости

Итак, зададимся вопросом: чему равна компонента вектора силы ( mathbf ) вдоль наклонной плоскости? Теперь мы знаем, что угол между вектором силы ( mathbf ) и наклонной плоскостью равняется 90°-​ ( theta ) ​. Значит, компонента вектора силы вдоль наклонной плоскости ( F_ ) равна:

Если вы добросовестно учили тригонометрию, то вам наверняка должно быть известно (а если нет, то обратитесь к главе 2), что:

(Часто это знать совсем не обязательно, и может сгодиться предыдущее уравнение.)

Полученное выражение можно легко проверить следующим образом. Когда ​ ( theta ) ​ стремится к 0°, то значение компоненты силы вдоль наклонной плоскости ( F_ ) стремится к 0, поскольку наклонная плоскость стремится к горизонтальному положению. А когда ​ ( theta ) ​ стремится к 90°, то значение компоненты силы вдоль наклонной плоскости ( F_ ) стремится к ​ ( F_g ) ​ поскольку наклонная плоскость стремится к вертикальному положению. Итак, если вдоль наклонной плоскости на тележку с массой 800 кг действует сила ​ ( F_gsintheta ) ​, то каким будет ускорение тележки? Это легко определить по известной формуле:

Задача упрощается, если вспомнить, что ​ ( F_g=mg ) ​ и тогда:

Итак, теперь нам известно, что ускорение тележки вдоль наклонной плоскости равно ​ ( a=gsintheta ) ​. Это соотношение справедливо для любого объекта, ускоряющегося под действием силы тяжести, если не учитывать силы трения.

Вычисляем скорость вдоль наклонной плоскости

Логично было бы поинтересоваться: а какова скорость тележки в конце наклонной плоскости? Для этого нам потребуется следующее уравнение, которое было выведено в главе 3:

Поскольку начальная скорость ​ ( v_0 ) ​ = 0, а длина наклонной плоскости ​ ( s ) ​ = 5 м, то получим:

Итак, скорость тележки в конце наклонной плоскости ( v_1 ) = 7 метров в секунду. Хотя это не такая уж и большая скорость для автомобиля, но все же не рекомендуется проводить такие эксперименты в домашних условиях. Имейте в виду, что на самом деле скорость будет несколько ниже, поскольку часть энергии расходуется на вращение колес, движение других частей автомобиля, трение и т.д.

Разбираемся с ускорением

Блиц-вопрос: а какую скорость в конце наклонной плоскости приобретет кубик льда при скольжении без трения? Ответ: он будет иметь такую же скорость, что и тележка в предыдущем примере, т.е. 7 м/с. Ускорение любого объекта, движущегося без трения вдоль наклонной плоскости под углом ​ ( theta ) ​, равно ​ ( gsintheta ) ​. Как видите, имеет значение не масса объекта, а компонента ускорения свободного падения вдоль наклонной плоскости. Если нам известно ускорение движения кубика льда и пройденное расстояние ​ ( s ) ​, то получим значение скорости по известной формуле:

Итак, масса не входит в формулу для определения конечной скорости.

Преодолеваем трение

Трудно представить себе повседневную жизнь без трения. Без трения автомобили не могли бы ездить, люди — ходить, а руки — брать любые предметы. Трение создает проблемы, но без него жизнь была бы просто невозможной.

Трение возникает из-за взаимодействия между поверхностными неровностями. Поверхность состоит из множества микроскопических выступов и впадин. При соединении двух поверхностей эти выступы одной поверхности и впадины другой поверхности сцепляются и препятствуют свободному проскальзыванию.

Допустим, что ваши сбережения хранятся в виде огромного золотого слитка, который показан на рис. 6.3, и некий злоумышленник задумал украсть его, но не может нести такой огромный слиток в руках, а может только тащить его волоком. Этот воришка стремится приложить силу к слитку, чтобы ускорить его и сбежать от преследующей его полиции. Однако благодаря силе трения вор не сможет развить большого ускорения.

Определим количественно влияние силы трения на движение объектов. Результирующая сила на слиток и создаваемое ею ускорение определяется как разность приложенной силы ​ ( F_п ) ​ и силы трения ​ ( F_ <трение>) ​ вдоль оси X:

Эта формула выглядит очень просто, но как определить силу трения? Как будет показано ниже, она зависит от нормальной силы.

Вычисляем силу трения и нормальную силу

Сила трения ( F_ <трение>) всегда противодействует приложенной силе, которая вызывает движение. Причем сила трения пропорциональна приложенной силе.

Как показано на рис. 6.3, слиток золота давит на горизонтальную поверхность с силой, равной весу слитка, ​ ( mg ) ​. А поверхность с той же силой действует на слиток. Эту силу называют нормальной силой (или силой нормального давления), ​ ( F_н ) ​.(Нормальной называется компонента силы со стороны поверхности, направленная по нормали к поверхности, т.е. перпендикулярно к поверхности.) Нормальная сила по величине не всегда совпадает с силой тяжести, поскольку нормальная сила всегда перпендикулярна поверхности, по которой движется объект. Иначе говоря, нормальная сила — это сила взаимодействия поверхностей разных объектов, и чем она больше, тем сильнее трение.

В примере на рис. 6.3 слиток скользит вдоль горизонтальной поверхности, поэтому нормальная сила равна весу объекта, т.е. ​ ( F_н=mg ) ​ Итак, у нас есть нормальная сила, которая равна силе давления слитка на горизонтальную поверхность. Для чего она нам нужна? Для определения силы трения.

Разбираемся с коэффициентом трения

Сила трения определяется характеристиками поверхностей соприкасающихся материалов. Как физики теоретически описывают их? Никак. У физиков есть множество общих уравнений, которые предсказывают общее поведение объектов, например ​ ( sum!F=ma ) ​ (см. главу 5). Однако у физиков нет полного теоретического понимания механизмов взаимодействия поверхностей материалов. Поэтому поверхностные характеристики материалов известны, в основном, из опыта.

А из опыта известно, что нормальная сила непосредственно связана с силой трения. Оказывается, что с большой точностью эти две силы пропорциональны друг другу и их можно связать с помощью константы ​ ( mu ) ​ следующим образом:

Согласно этому уравнению, чтобы определить силу трения, нужно умножить нормальную силу на некую постоянную величину, т.е. константу ​ ( mu ) ​. Такая константа называется коэффициентом трения, и именно она характеризует свойства сцепления шероховатостей данных поверхностей.

Величина коэффициента трения находится в диапазоне от 0 до 1. Значение 0 возможно только в идеализированном случае, когда трение отсутствует вообще. А значение 1 соответствует случаю, когда сила трения максимальна и равна нормальной силе. Это значит, что максимальная сила трения для автомобиля не может превышать его веса.

Обратите внимание, что уравнение ​ ( F_<трение>=mu F_н ) ​ не является соотношением между векторами, поскольку эти векторы направлены в разные стороны. Например, на рис. 6.3 они перпендикулярны друг другу. Действительно, нормальная сила ( mathbf ) всегда перпендикулярна поверхности, а сила трения ​ ( mathbf> ) ​ — параллельна. Эти направления определяются их природой: нормальная сила ( mathbf ) определяет степень сжатия поверхностей, а сила трения ( mathbf> ) — степень противодействия скольжению вдоль поверхностей.

Сила трения не зависит от площади соприкосновения двух поверхностей. Это значит, что слиток с той же массой, но вдвое длиннее и вдвое ниже исходного будет испытывать точно такую же силу трения при скольжении по поверхности. При этом увеличивается вдвое площадь соприкосновения, но уменьшается вдвое давление, т.е. величина силы, которая приходится на единицу площади.

Итак, мы получили предварительные сведения и готовы вычислить силу трения? Не так быстро. Оказывается, что коэффициент трения бывает двух типов.

Знакомимся со статическим и кинетическим трением

Два разных коэффициента трения соответствуют двум разным типам трения: статическому трению (или трению покоя) и кинетическому трению (или трению скольжения).

Дело в том, что эти типы трения соответствуют двум разным физическим процессам. Если две поверхности не движутся относительно друг друга, то на микроскопическом уровне они взаимодействуют более интенсивно, и этот случай называется трением покоя. А когда поверхности уже скользят относительно друг друга, то микроскопические неровности не успевают вступить в интенсивное взаимодействие, и этот случай называется трением скольжения. На практике это значит, что для каждого из этих двух типов трения используются свои коэффициенты трения: коэффициент трения покоя ​ ( mu_п ) ​ и коэффициент скольжения ( mu_с ) .

Изучаем статическое трение

Трение покоя сильнее трения скольжения, т.е. коэффициент трения покоя ( mu_п ) больше коэффициента трения скольжения ( mu_с ) . Это можно упрощенно объяснить следующим образом. В состоянии покоя соприкасающиеся поверхности интенсивно взаимодействуют на микроскопическом уровне, а при скольжении поверхности успевают вступить в интенсивное взаимодействие только на более крупном макроскопическом уровне.

Трение покоя возникает тогда, когда нужно привести в движение покоящийся объект. Именно такую силу трения нужно преодолеть для начала скольжения объекта.

Предположим, что в примере на рис. 6.3 коэффициент трения покоя между слитком и поверхностью равен 0,3, а масса слитка равна 1000 кг (очень приличный слиток). Какую силу должен приложить воришка, чтобы сдвинуть слиток? Из предыдущих разделов нам уже известно, что:

Поскольку поверхность горизонтальна, то нормальная сила направлена противоположно силе тяжести слитка и имеет ту же величину:

где ​ ( m ) ​ — масса слитка, a ​ ( g ) ​ — ускорение свободного падения, вызванное силой притяжения со стороны Земли. Подставляя численные значения, получим:

Итак, воришке потребуется приложить силу 2940 Н, чтобы сдвинуть с места неподвижный слиток. Довольно большая сила! А какая сила потребуется ему, чтобы поддерживать скольжение слитка? Для ответа на этот вопрос нужно рассмотреть трение скольжения.

Поддерживаем движение вопреки трению скольжения

Сила трения скольжения, возникающая из-за скольжения двух соприкасающихся поверхностей, не так велика, как сила трения покоя. Но это совсем не значит, что коэффициент трения скольжения можно легко вычислить теоретически, даже если нам известен коэффициент трения покоя. Оба коэффициента трения приходится определять из опыта.

Именно из опыта известно, что трение покоя больше трения скольжения. Представьте себе, что вы разгружаете неподвижный ящик на наклонной плоскости, но он вдруг начинает скользить вниз. Достаточно заблокировать его движение ногой и с большой вероятностью ящик останется в состоянии покоя, если аккуратно убрать ногу. Именно так, в состоянии покоя, проявляется трение покоя, а в процессе движения ящика — трение скольжения.

Пусть слиток на рис. 6.3 имеет массу 1000 кг, а коэффициент трения скольжения ​ ( mu_c ) ​ равен 0,18. Какую силу должен приложить воришка, чтобы сдвинуть с места неподвижный слиток? Для ответа на этот вопрос нужно воспользоваться следующей формулой:

Подставляя численные значения, получим:

Воришке потребуется приложить силу 1764 Н, чтобы поддерживать скольжение слитка. Не такая уж и маленькая сила, если, конечно, воришке не помогают его верные друзья. Однако это не так уж и легко, и полиция быстро сможет догнать этого воришку. Зная законы физики, полицейские вряд ли захотят прилагать лишние усилия: “Слиток-то мы нашли, а вот домой тащите его сами”.

Тянем груз в гору и боремся с трением

В предыдущих примерах со слитком описывалось трение на горизонтальной поверхности. А как определить силу сопротивления со стороны трения на наклонной плоскости?

Допустим, что, собираясь на рыбалку, вы решили захватить с собой холодильник массой 100 кг. Единственный способ погрузить его в багажник автомобиля — это втащить холодильник по наклонной плоскости, как показано на рис. 6.4. Пусть наклонная плоскость расположена под углом 30°, коэффициент трения покоя равен 0,2, а коэффициент трения скольжения — 0,15. Хорошая новость заключается в том, что вам помогают два друга, а плохая — в том, что каждый из вас способен приложить силу не более 350 Н.

Ваши друзья растеряны? “Не стоит беспокоиться, немного физики — и все будет в порядке”, — можете ответить им вы, доставая калькулятор. Итак, нам нужно вычислить минимальную силу, которую нужно приложить, чтобы втащить холодильник вверх по наклонной плоскости в багажник автомобиля вопреки силе трения и силе тяжести.

Вычисляем компоненту силы тяжести

Для этого нужно внимательно изучить схему на рис. 6.4. Сила тяжести действует на холодильник и направлена вертикально вниз. Сумма углов треугольника, образованного вектором силы тяжести, наклонной плоскостью и ее основанием, равна 180°. Угол между вектором силы тяжести и основанием наклонной плоскости равен 90°, а угол между наклонной плоскостью и ее основанием — ​ ( theta ) ​. Поэтому угол между наклонной плоскостью и вектором силы тяжести равен:

Компонента силы тяжести, действующая вдоль наклонной плоскости, равна:

Таким образом, минимальная сила, с которой нужно толкать холодильник вверх по наклонной плоскости, равна сумме силы трения, ​ ( F_ <трение>) ​, и этой компоненты ( F_ ) , т.е.:

Определяем силу трения

Следующий вопрос: чему равна сила трения, ( F_ <трение>) ? Какой коэффициент трения нужно использовать для ее определения: покоя или скольжения? Поскольку коэффициент трения покоя больше коэффициента трения скольжения, то для оценки минимально необходимой силы имеет смысл учесть коэффициент трения покоя. Ведь после того как холодильник удастся сдвинуть с места, для скольжения придется прикладывать меньшую силу. Итак, с учетом коэффициента трения покоя, получим для силы трения

Для определения этой силы трения нам потребуется вычислить нормальную силу, ( F_н ) (более подробно эта сила описывается выше в этой главе). Она равна компоненте силы тяжести, которая направлена перпендикулярно (т.е. по нормали, откуда и происходит ее название) к наклонной плоскости. Как мы уже выяснили, угол между наклонной плоскостью и вектором силы тяжести равен 90°-​ ( theta ) ​(рис. 6.5).

С помощью тригонометрических соотношений (см. главу 2) получим:

Чтобы проверить справедливость этого выражения, попробуйте устремить угол ​ ( theta ) ​ к нулю, при котором нормальная сила ​ ( F_н ) ​ становится равной ​ ( mg ) ​, что и следовало ожидать. Теперь получаем:

После подстановки численных значений получим:

Итак, три человека должны приложить минимально необходимую силу 660 Н, т.е. по 220 Н каждый, что меньше максимально возможной силы 350 Н. С радостным призывом “Приступим!” вы приступаете к работе, втаскиваете холодильник на самый верх наклонной плоскости. Допустим, что из-за несогласованности действий кто-то из вас перестал прикладывать силу. Как результат, холодильник после непродолжительной остановки неожиданно заскользил вниз, а после достижения основания продолжил движение по полу до полной остановки.

Вычисляем путь скольжения холодильника до полной остановки

Допустим, что наклонная плоскость и пол имеют одинаковые коэффициенты трения скольжения. Каким будет путь скольжения холодильника до полной остановки? Пусть сначала холодильник скользит из состояния покоя до основания наклонной плоскости длиной 3 м, как показано на рис. 6.6. Во время такого скольжения холодильник разгоняется и вполне может столкнуться с автомобилем на расстоянии 7,5 м. О, Боже! Неужели они столкнутся? Нужно немедленно достать калькулятор и приступить к расчетам.

Вычисляем ускорение скольжения

При скольжении вниз действующие на холодильник силы направлены иначе, чем при скольжении вверх. Теперь вы и ваши друзья уже не прилагают свои силы, а холодильник скользит только под действием компоненты силы тяжести, направленной вдоль наклонной плоскости. А ей противодействует лишь сила трения. Чему же равна результирующая сумма этих сил? Из предыдущих разделов уже известно, что компонента силы тяжести вдоль наклонной плоскости равна:

А нормальная сила равна:

Это значит, что сила трения скольжения равна:

Результирующая сила, которая действует на холодильник в направлении движения и определяет его ускорение, равна:

Обратите внимание на то, что сила трения, ​ ( F_ <трение>) ​, имеет отрицательный знак, т.е. она направлена противоположно компоненте силы тяжести вдоль наклонной плоскости, которая приводит в движение холодильник. После подстановки численных значений получим:

Поскольку масса холодильника равна 100 кг, то он скользит с ускорением 363 Н/100 кг = 3,63 м/с 2 вдоль наклонной плоскости длиной 3 м. Для вычисления конечной скорости холодильника, ​ ( v ) ​, в конце наклонной плоскости нужно использовать следующую известную нам формулу:

После извлечения квадратного корня и подстановки численных значений получим:

Такой будет скорость холодильника в конце наклонной плоскости.

Вычисляем путь скольжения по полу

Как на основе данных, полученных в предыдущем разделе, определить путь скольжения холодильника по полу? Столкнется ли холодильник с автомобилем?

Итак, нам известно, что холодильник начинает движение по полу со скоростью 4,67 м/с. Вопрос: какое расстояние он пройдет до полной остановки? Теперь в горизонтальном направлении на него действует только сила трения, а компонента силы тяжести по горизонтали равна нулю. Поэтому холодильник постепенно замедляется и рано или поздно остановится. Но уцелеет ли при этом стоящий поодаль автомобиль? Как обычно, сначала вычисляем суммарную силу ​ ( F ) ​, действующую на холодильник в направлении движения и определяющую его ускорение. В данном случае она равна силе трения:

Поскольку холодильник движется вдоль горизонтальной поверхности, то нормальная сила ​ ( F_н ) ​ равна силе тяжести ( F_g ) , действующей на холодильник:

т.е. суммарная сила равна:

После подстановки численных значений получим:

Именно такая сила сопротивления действует на холодильник и… терроризирует всю округу! Итак, насколько длинным будет тормозной путь холодильника? Подставим численные значения и получим:

Здесь отрицательный знак обозначает замедление холодильника (см. главу 2).

найдем тормозной путь холодильника:

Поскольку конечная скорость ​ ( v_1 ) ​, равна 0, то эта формула упрощается и принимает вид:

Вот это да! Холодильник проедет расстояние 7,4 м и остановится всего в 10 см от автомобиля, который находится на расстоянии 7,5 м от основания наклонной плоскости. Можно расслабиться и понаблюдать за вашими друзьями, которые охвачены паникой и с ужасом в глазах ожидают столкновения холодильника и автомобиля.

Как гравитация влияет на свободное падение объектов

В главе 7 сила гравитационного притяжения (или сила тяжести) описывается в космическом масштабе, а здесь она рассматривается только вблизи поверхности Земли. В физике часто встречаются задачи с учетом силы тяжести. Этот раздел посвящен тому, как сила тяжести влияет на свободное падение объектов, и его следует рассматривать, как переходный между материалом предыдущей главы и материалом главы 7.

Стреляем вверх: максимальная высота

Зная ускорение свободного падения и начальную скорость объекта, можно легко вычислить дальность его полета. Эти знания могут пригодиться при подготовке праздничных фейерверков!

Предположим невероятное: на день рождения друзья подарили вам пушку, способную разгонять ядро весом 10 кг до начальной скорости 860 м/с. С изумлением рассматривая ее, гости начали спорить: а на какую максимальную высоту эта пушка способна выстрелить? Поскольку вы уже владеете всеми необходимыми знаниями, то можете быстро дать ответ на этот вопрос.

Нам известна начальная скорость ядра, ​ ( v_0 ) ​, и ускорение свободного падения ​ ( g ) ​ под действием силы тяжести. Как определить максимальную высоту подъема ядра? В точке максимального подъема ядра его скорость будет равна нулю, а затем оно начнет обратное движение вниз. Следовательно, для вычисления максимальной высоты подъема ядра, ​ ( s ) ​, можно использовать следующую формулу, в которой конечная скорость ​ ( v_1 ) ​ равна нулю:

Подставляя численные значения для начальной скорости ​ ( v_0 ) ​ = 860 м/с 2 , ускорения свободного падения под действием силы тяжести ​ ( g ) ​ = —9,8 м/с 2 (минус обозначает направление ускорения, противоположное направлению перемещения), получим:

Ого! Ядро улетит на высоту 38 км. Совсем неплохо для пушки, подаренной на день рождения. Интересно, а сколько же времени придется его ждать обратно?

Время подъема ядра

Итак, сколько времени потребуется для того, чтобы ядро поднялось на максимальную высоту? В примере из главы 4, где мяч для игры в гольф падал с вершины обрыва, для вычисления дальности его полета использовалось следующее уравнение:

Однако это уравнение представляет собой всего один из многих возможных вариантов поиска ответа на заданный вопрос.

Нам известно, что в точке максимального подъема скорость ядра равна 0. Поэтому для определения времени полета до максимальной высоты можно использовать следующее уравнение:

Поскольку ​ ( v_1 ) ​ = 0 и ​ ( a ) ​ = ​ ( -g ) ​, то:

Иначе говоря, получим:

После подстановки численных значений получим:

Итак, ядру потребуется 88 с, чтобы достичь максимальной высоты. А каково общее время полета?

Общее время полета

Сколько времени потребуется ядру, чтобы достичь максимальной высоты 38 км и вернуться обратно к пушке, если на подъем ему потребовалось 88 с? Общее время полета вычислить очень просто, поскольку обратный путь вниз симметричен прямому пути вверх. Это значит, что скорость ядра в каждой точке обратного пути вниз равна по величине и имеет противоположное направление по сравнению с прямым путем вверх. Поэтому время падения равно времени подъема и общее время полета равно удвоенному времени подъема:

Итак, общее время полета равно 176 с, или 2 минуты и 56 секунд.

Стреляем под углом

В предыдущих разделах пушка стреляла вертикально вверх. Попробуем теперь поразить цель, стреляя ядром из пушки под углом, как показано на рис. 6.7.

Разбиваем движение ядра на компоненты

Как характеризовать движение ядра при стрельбе под углом? Поскольку любое движение всегда можно разбить на компоненты по осям X и Y, а в данном примере сила притяжения действует только вдоль оси Y, то задача упрощается. Разобьем начальную скорость на компоненты (подробнее об этом рассказывается в главе 4):

Эти компоненты независимы, а сила притяжения действует только в направлении оси Y. Это значит, что компонента ​ ( v_x ) ​ остается постоянной, а меняется только компонента ​ ( v_y ) ​:

Теперь легко определить координаты ядра в любой момент. Например, координата ядра по оси X выражается формулой:

Поскольку сила тяжести влияет на движение ядра по вертикали, то координата ядра по оси Y выражается формулой:

Из предыдущего раздела нам уже известно, что общее время полета ядра по вертикали равно:

Теперь, зная время, можно легко определить дальность полета ядра по оси X:

Итак, для вычисления дальности полета ядра по горизонтали нужно знать начальную скорость ядра, ​ ( v_0 ) ​, и угол, ​ ( theta ) ​, под которым сделан выстрел.

Определяем максимальную дальность полета ядра

При каком угле выстрела ( theta ) ядро улетит на максимальное расстояние по горизонтали? Из тригонометрии известно, что ​ ( 2sinthetacostheta=sin2theta ) ​.

и расстояние ​ ( s ) ​ будет максимальным при максимальном значении ​ ( sin2theta=1 ) ​, т.е. при ​ ( theta ) ​ = 45°.

Совсем неплохо для пушки, подаренной на день рождения!

Трение

В жизни все выглядит немного по-другому, т.к. мы постоянно сталкиваемся с трением. И это хорошо! Не будь трения — как бы мы жили? Ведь тогда нельзя было ни ходить, ни взять что-то в руки…

Какие же силы действуют на тело, которое мы пытаемся сдвинуть с места?

Сила трения пропорциональна приложенной силе и противодействует ей. Шар давит на поверхность с силой m·g. А поверхность с той же силой действует на шар. Эту силу называют нормальной — Fн.

Нормальная сила всегда направлена перпендикулярно к поверхности

В нашем случае Fн = m·g, т.к. поверхность горизонтальна. Но, нормальная сила по величине не всегда совпадает с силой тяжести.

Нормальная сила — сила взаимодействия поверхностей соприкасающихся тел, чем она больше — тем сильнее трение.

Нормальная сила и сила трения пропорциональны друг другу:

0 180° — 90° — α = 90° — α

Составляющие силы тяжести вдоль наклонной плоскости:

Необходимая сила для поднятия шара:

Необходимо определить силу трения Fтр. С учетом коэффициента трения покоя:

Вычисляем нормальную силу Fнорм, которая равна составляющей силы тяжести, перпендикулярно направленной к наклонной плоскости. Мы уже знаем, что угол между вектором силы тяжести и наклонной плоскостью равен 90° — α.

Fнорм = mgsin(90° — α) = mgcosα
F = mgsinα + μmgcosα

F = 1·9,8·sin30° + 0,1·1·9,8·cos30° = 4,9 + 0,85 = 5,75 Н

Нам потребуется к шару приложить силу в 5,75 Н для того, чтобы закатить его на вершину наклонной плоскости.

Задача №2: определить как далеко прокатится шар массой m = 1 кг по горизонтальной плоскости, скатившись по наклонной плоскости длиной 10 метров при коэффициенте трения скольжения μ = 0,05

Силы, действующие на скатывающийся шар, приведены на рисунке.

Составляющая силы тяжести вдоль наклонной плоскости:

Fн = mgsin(90° — α) = mgcos(90° — α)

Сила трения скольжения:

Fтрения = μFн = μmgsin(90° — α) = μmgcosα

F = Fg — Fтрения = mgsinα — μmgcosα

F = 1·9,8·sin30° — 0,05·1·9,8·0,87 = 4,5 Н

F = ma; a = F/m = 4,5/1 = 4,5 м/с 2

Определяем скорость шара в конце наклонной плоскости:

V 2 = 2as; V = &38730;2as = &38730;2·4,5·10 = 9,5 м/с

Шар заканчивает движение по наклонной плоскости и начинает движение по горизонтальной прямой со скоростью 9,5 м/с. Теперь в горизонтальном направлении на шар действует только сила трения, а составляющая силы тяжести равна нулю.

F = μFн = μFg = μmg = 0,05·1·9,8 = -0,49 Н

Знак минус означает, что сила направлена в противоположную сторону от движения. Определяем ускорение замедления шара:

a = F/m = -0,49/1 = -0,49 м/с 2

Тормозной путь шара:

Поскольку мы определяем путь шара до полной остановки, то V1=0:

s = (-V0 2 )/2a = (-9,5 2 )/2·(-0,49) = 92 м

Наш шарик прокатился по прямой целых 92 метра!

Если вам понравился сайт, будем благодарны за его популяризацию :) Расскажите о нас друзьям на форуме, в блоге, сообществе. Это наша кнопочка:

Код кнопки:
Политика конфиденциальности Об авторе

Понравилась статья? Поделить с друзьями:
  • Четыре способа как найти человека
  • Как правильно найти сонную артерию
  • Как найти объем поверхности четырехугольной пирамиды
  • Как найти блоггера в ютубе по имени
  • Как правильно составить письмо с примерами